You are on page 1of 75

1 2

Laurel v. Abrogar BICERRA VS. TENEZA


G.R. No. 155076, January 13, 2009
Facts: FACTS:

Philippine Long Distance Telephone Company (PLDT) filed a The Bicerras were the owners of the house, worth P200.00, built
complaint for theft under Article 308 of the Revised Penal Code on and owned by them and situated in the said municipality
against Baynet Co., Ltd. (Baynet) for stealing its business. PLDT Lagangilang. In January 1957, the Tenezas forcibly demolished
alleged that Baynet offered phone cards to people in Japan to call the house, claiming to be the owners thereof. The materials of the
their friends and relatives in the Philippines using PLDT's facilities house, after it was dismantled, were placed in the custody of the
and equipment. barrio lieutenant of the place.The Bicerras filed a complaint
claiming actual damages of P200, moral and consequential
Issue: damages amounting to P600, and the costs. The CFI Abra
dismissed the complaint claiming that the action was within the
Whether or not the PLDT's business of providing exclusive (original) jurisdiction of the Justice of the Peace Court
telecommunication services is a personal property under Article of Lagangilang, Abra.
308 of the Revised Penal Code.
ISSUE: W/N The action involves title to real property, as
Held:
appellants contend, and therefore is cognizable by the Court of
No, PLDT's business of providing telecommunication services is First Instance.
not a personal property under Article 308 of the Revised Penal
Code. HELD: Yes, the action involves a real property.

Term “personal property” as used in Art. 308 of RPC should be A house is classified as immovable property by reason of its
interpreted in the context of the Civil Code's definition of real and adherence to the soil on which it is built (Art. 415, par. 1, Civil
personal property. Consequently, any personal property, tangible Code). This classification holds true regardless of the fact that the
or intangible, corporeal or incorporeal, capable of appropriation house may be situated on land belonging to a different owner. But
may be the subject of theft so long as the same is not included in once the house is demolished, as in this case, it ceases to exist
the enumeration of Real Properties under the Civil Code. The as such and hence its character as an immovable likewise
ceases. It should be noted that the complaint here is for recovery
only requirement for personal property to capable of theft, is that
of damages. This is the only positive relief prayed for by
it be subject to appropriation.
appellants. To be sure, they also asked that they be declared
PLDT's business is intangible and cannot be taken by another owners of the dismantled house and/or of the materials. However,
and not the proper subjects of theft because they are without form such declaration in no wise constitutes the relief itself which if
or substance.


granted by final judgment could be enforceable by execution, but
is only incidental to the real cause of action to recover damages.

The order appealed from is affirmed. The appeal having been


admitted in forma pauperis, no costs are adjudged.

3. PUNZALAN VS. VDA. DE LACSAMANA, 121 SCRA 331 (1983)

Facts:
PNB filed a Motion to Dismiss on the ground that venue was
improperly laid considering that the building was real property
under article 415 (1) of the New Civil Code and therefore section
2(a) of Rule 4 of the Rules of Court should apply. Petitioner
Punzalan contended that the action for annulment of deed of sale
with damages is in the nature of a personal action, which seeks to
recover not the title nor possession of the property but to compel
payment of damages, which is not an action affecting title to real
property.

Issue: W/N the building or warehouse is a real property?

Ruling:
The warehouse claimed to be owned by petitioner is an
immovable or real property as provided in article 415(l) of the Civil
Code. Buildings are always immovable under the Code. A
building treated separately from the land on which it stood is
immovable property and the mere fact that the parties to a
contract seem to have dealt with it separate and apart from the
land on which it stood in no wise changed its character as
immovable property.


4 5

G.R. No. L-50466 May 31, 1982

CALTEX (PHILIPPINES) INC., petitioner,


vs.
CENTRAL BOARD OF ASSESSMENT APPEALS and CITY
ASSESSOR OF PASAY, respondents.

FACTS:
This case is about the realty tax on machinery and equipment
installed by Caltex (Philippines) Inc. in its gas stations located on
leased land.
The said machines and equipment are loaned by Caltex to gas
station operators under an appropriate lease agreement or
receipt. Under the lease contract, the operators are bound to
return the equipment in good condition upon demand and that
Caltex retains ownership thereof during the term of the lease.
The Central Board of Assessment Appeals held that the
machines and equipment are real property tax within the meaning
of sections 3(k) & (m) and 38 of the Real Property Tax Code and
that the definitions of real property and personal property in
articles 415 and 416 of the Civil Code are not applicable to this
case.
Caltex filed a petition for certiorari praying to set aside the
Board’s decision and for a declaration that the said machines and
equipment are personal property not subject to realty tax.
ISSUE:
Whether or not the machines and equipment are real property
subject to realty tax.
HELD:


Yes, the machines and equipment are real property subject CASE No. 06
to realty tax. Said equipment and machinery, as appurtenances
MERALCO vs. CBAA, G.R. No. L-46245, May 31, 1982
to the gas station building or shed owned by Caltex (as to which it
is subject to realty tax) and which fixtures are necessary to the Facts:
operation of the gas station, for without them the gas station Meralco Securities installed from Batangas to Manila, a pipeline
would be useless, and which have been attached or affixed system consisting of cylindrical steel pipes joined together and
permanently to the gas station site or embedded therein, are buried not less than one meter below the surface along the
taxable improvements and machinery within the meaning of the shoulder of the public highway. The pipes are embedded in the
Assessment Law and the Real Property Tax Code. soil while the valves are welded to the pipes so as to make the
pipeline system one single piece of property from end to end.
Improvements on land are commonly taxed as realty even though Pursuant to the Assessment Law, Commonwealth Act No. 470,
for some purposes they might be considered personalty. the provincial assessor of Laguna treated the pipeline as real
Petition for certiorari is dismissed. property and issued Tax Declarations.

Issue:
Whether or not the Meralco Securities Pipeline System in Laguna
is a subject to a realty tax.

Held: Meralco Securities Pipeline System in Laguna is subject to


a realty tax. CBAA reasoned out that the pipes are machinery or
improvements and regarded as realty because they are
constructions adhered to the soil. It is attached to the land in such
a way that it cannot be separated therefrom without dismantling
the steel pipes which are welded to the pipeline. In so far as the
pipeline uses valves, pumps and control devices to maintain the
flow of the oil, it is in a sense a machinery within the meaning of
the Real Property Tax Code.

Thus, the Court dismiss the petition and the questioned decision
and resolution of the lower court is affirmed.



7 1. Yes. Petition for annulment of judgment was the proper
remedy available to the bank. Such bank was deprived of its
METROPOLITAN BANK, & TRUST COMPANY vs. Hon. duly registered property right without due process of the law
FLORO T. ALEJO, Presiding Judge of Branch 172 RTC by not including the petitioner as defendant to the civil case by
Valenzuela; Sy Tan Se even if he was an indispensable party.

FACTS: As security for the payment of the loans obtained from


Metropolitan Bank and Trust Company by the spouses Raul and 2. Yes. The judgment of the trial court should also be declared
Cristina Acampado, a Real Estate Mortgage over a parcel of null and void because the bank, which is an indispensable
Nullity of land was executed by same spouses. Subsequently a party, was not impleaded in the civil case. The absence of an
Complaint for Declaration of the TCT of the spouses was filed by indispensable party renders all subsequent actuations of the
Sy Tan Se in the RTC of Valenzuela. The bank was not made a court null and void, for want of authority to act, not only as to
party to the said civil case(complaint for declaration of nullity of the absent parties but even as to those present.
TCT.) They weren’t notified as well. The spouses defaulted in the
payment of their loan and extrajudicial foreclosure proceedings Case Number 8:
were initiated. The bank submitted the highest and winning bid. A
certificate of sale was issued in their favor. Chavez v. Public Estates Authority
415 SCRA 403 (2003)
When they were about to get their TCT from the Register of
Deeds, petitioner was informed of the existence of the decision in FACTS:
the aforementioned civil case (complaint for declaration of nullity The Register of Deeds of Paranaque issued TCT’s in the name of
of TCT) declaring the Spouses Acampados’s TCT null and void. PEA, as successor of CDCP, covering the 3 reclaimed islands
known as the “Freedom Islands” located in the southern portion of
The bank filed with the CA a petition for the annulment of the the Manila-Cavite Coastal Road. On April 25, 1995 , PEA entered
RTC Decision. The CA dismissed their petition and ruled that the into a Joint Venture Agreement with AMARI, a private
bank should have filed a petition for relief from judgment or an corporation, to develop the Freedom Islands and to reclaim an
action for quieting of title. additional 250 hectares of submerged areas surrounding the
islands. With this, PEA obliged itself to transfer to AMARI 70% of
ISSUES: PEA’s proportionate share in the reclaimed areas

1. Whether or not a petition for annulment of judgment is the ISSUE:


proper remedy available to the bank. Is the Joint Venture Agreement, which will transfer the reclaimed
2. Whether or not the judgment of the trial court (declaring lands, valid?
the Spouses Acampados TCT null and void) should be
declared null and void RULING:
No. Under Sec.2, Article XII of the Constitution, the foreshore and
HELD : submerged areas of Manila Bay are part of lands of the public


domain, and consequently owned by the State. As such, these The RTC rendered a ruling granting the respondents’ Application
areas shall not be alienated, unless they are classified as for Original Registration of Lot 3. The Court of Appeals affirmed
agricultural lands of the public domain. the RTC’s decision on appeal.
The mere reclamation of these areas by PEA does not convert
ISSUE:
these into alienable or disposable lands of the public domain.
There must be a law or presidential proclamation officially or WON the Court of Appeals erred in affirming the RTC ruling
classifying these reclaimed lands as alienable or disposable and granting original registration of Lot 3 in favor of the respondents.
open to disposition or concession. Moreover, these reclaimed
lands cannot be classified as alienable or disposable if the law RULING:
has reserved them for some public or quasi-public use. Absent We grant the petition.
these two official acts – classification as alienable and a
declaration that these lands are not needed for public service, Regalia simply means that the State is the original proprietor of all
lands reclaimed by PEA remain inalienable lands of public lands and, as such, is the general source of all private titles.
domain that cannot be transferred even by Joint Venture Thus, pursuant to this principle, all claims of private title to land,
Agreement. Hence, The Joint Venture Agreement is save those acquired from native title, must be traced from some
unconstitutional. grant, whether express or implied, from the State. Absent a clear
showing that land had been let into private ownership through the
State’s imprimatur, such land is presumed to belong to the State.
9) Republic v. Santos
Being an unregistered land, Lot 3 is therefore presumed as land
FACTS: belonging to the State. It is basic that those who seek the entry of
such land into the Torrens system of registration must first
The respondents purchased three (3) parcels of unregistered establish that it has acquired valid title thereto as against the
land. After the said purchase, the respondents caused the survey State, in accordance with law.
and consolidation of the parcels of land into a single lot—“Lot 3”.
They then filed an Application for Original Registration of Lot 3.
They alleged that their predecessors-in-interest i.e., the previous 10. VILLARICO VS SARMIENTO, 442 SCRA 110
owners of the parcels of land making up Lot 3, have been in
“continuous, uninterrupted, open, public and adverse” possession
of the said parcels “since time immemorial.” Facts:
The government, through the Office of the Solicitor General, filed Villarico is an owner of a lot that is separated from the
the lone opposition to the respondents’ application. It insists that
Ninoy Aquino Avenue highway by government land. Sarmiento
Lot 3 still forms part of the public domain and, hence, not subject
had a building constructed on a portion of the government land. In
to private acquisition and registration. The government, however,
presented no further evidence to controvert the claim of the 1993, Villarico acquired a portion of the same area owned by the
respondents. government. He then filed an accion publiciana alleging that
respondents on the government land closed his right of way to
the NAA and encroached on a portion of his lot.


Are the parties of the Contract of Lease in Pari delicto?
Issue: Whether or not VIllarico has a right of way to the NAA. Ruling:
Under the 1987 Constitution specifically declares that all lands of
the public domain, waters, fisheries and other natural resources
Held:
belong to the State.18Included here are fishponds, which may not
No. It is not disputed in this case that the alleged right of way to be alienated but only leased, thus making the Contract of Lease
the lot belongs to the state or property of public dominion. void. We apply the maxim NEMO DAT QUOD NON HABET’ -
meaning ONE CANNOT GIVE WHAT HE DOES NOT HAVE.
Petitioners are applicants of the fishponds. Even if the State
Public use” means “use which is not confined to privileged grants them the lease, the law also prohibits sublease of the
individuals, fishponds.

but is open to the indefinite public. Public dominion property is The parties are in pari delicto or in equal fault and the law deny
outside the commerce of man. Hence, It cannot be burdened by a them aid for any claims against the other and courts leave them
voluntary easement of right of way in favor of the petitioner and as they are. However Article 1412 of the Civil Code provides the
petitioner cannot appropriate it for himself and he cannot claim exception.
any right of possession over it. "Art. 1412. If the act in which the unlawful or forbidden cause
consists does not constitute a criminal offense, the following rules
11. Menchavez vs. Teves
shall be observed:
Facts: "(1) When the fault is on the part of both contracting parties,
neither may recover what he has given by virtue of the contract,
On February 28, 1986 a Contract of Lease of a Fishpond was
or demand the performance of the other’s undertaking;
executed between the plaintiffs, as lessor and the respondent as
lessee. On June 2, 1988 the fishpond dikes constructed by "(2) When only one of the contracting parties is at fault, he cannot
respondent was demolished as ordered by Cebu RTC. As a recover what he has given by reason of the contract, or ask for
result, he filed a Complaint for damages alleging that petitioners the fulfillment of what has been promised him. The other, who is
violated their contract of lease specially the peaceful and not at fault, may demand the return of what he has given without
adequate enjoyment of property for the entire duration of the any obligation to comply with his promise."
contract. The RTC declared the Contract as void and declared
both parties in pari delicto. The CA however reversed the The petition is GRANTED.
decision. Thus the present appeal.
12. DACANAY V ASISTIO, JR
Issue:
[G.R. No. 93654 May 6, 1992]


are null and void for being contrary to law. Thus, City Mayor may
Facts: not infringe upon the vested right of the public to use city streets
for the purpose they were intended to serve, i. e. as areteries of
A municipal ordinance was passed designating certain city and travel for vehicles and pedestrians.
municipal streets, roads and open spaces as sites for flea
markets, one of which is Heroes del ’96 where herein petitioner Petition granted.
resides. A change in the administration of the City of Caloocan
caused the demolition of the market stalls. Stall owners filed
before the RTC an action for prohibition against city officials. The
court issued the writ prayed for, but subsequently dismissed the
petition and lifted the writ of preliminary injunction which it had
earlier issued after it was found that subject streets are of public
dominion, hence, outside the commerce of man. However, shortly
after the decision came out, the city administration in Caloocan
City changed hands and the policy of clearing and cleaning up
the city streets did not push through. As the stall holders
continued to occupy the land through the tolerance of the public
respondents, and in clear violation of the court decision
previously issued, Dacanay filed the present petition
for mandamus praying that the public respondents be ordered to
enforce the final decision which upheld the city mayor's authority
to order the demolition of market stalls and other pertinent laws.

Issue:
May public streets or thoroughfares be leased or licensed to
market stallholders by virtue of a city ordinance or resolution?
Held:

No. A public street is property for public use, hence, outside the
commerce of man and may not be the subject of lease or of any
other contract. The right of the public to use the city streets may
not be bargained away thru a contract. As the stallholders pay
fees to the City Government for the right to occupy portions of the
public street, the City Government, contrary to law, has been
leasing portions of the streets to them. Such leases or licenses


13 strictly construed against the grantee because it is a gratuitous
MANILA LODGE 761
VS donation of public money or resources, which resulted in an unfair
COURT OF APPEALS advantage to the grantee. In the case at bar, the area reclaimed
73 SCRA 162 would be filled at the expense of the Insular Government and
FACTS: without cost to the City of Manila. Hence, the letter of the statute
should be narrowed to exclude matters which, if included, would
The Philippine Commission enacted Act No. 1306 which defeat the policy of legislation.
authorized the City of Manila to reclaim a portion of Manila Bay.
The reclaimed area was to form part of the Luneta extension. The 14
act provided that the reclaimed area shall be the property of the
Republic VS. CA
City of Manila, and the city is authorized to set aside a tract of the
reclaimed land for a hotel site and to lease or to sell the same. 132 SCRA 514
Later, the City of Manila conveyed a portion of the reclaimed area Alluvium must be the exclusive work of nature. It has 3
to Petitioner. Then Petitioner sold the land, together with all the requirements: 1) that the deposit be gradual and imperceptible; 2)
improvements, to the Tarlac Development Corporation (TDC). through the current of the river; and 3) the land where the
accretion takes place is adjacent to the river bank. Deposits made
by human intervention are excluded.
ISSUE:

W/the subject property was patrimonial property of the City of FACTS:


Manila. The respondents (Tancinco’s) were registered owners of a parcel
of land in Bulacan, bordering on the Maycauayan and Bocaue
Rivers. They filed an application for the registration of three lots
HELD: adjacent to their fishpond, but because of the recommendation of
the Commissioner, they only pushed for the registration of two.
No, the petitions were denied for lack of merit. The court The RTC and CA granted the petition despite the opposition of
found it necessary to analyze all the provisions of Act No. 1360, the Bureau of Lands.
as amended, in order to unravel the legislative intent. The grant
made by Act No. 1360 of the reclaimed land to the City of Manila The respondents based their claim on accretions to their
is a grant of a “public” nature. Such grants have always been fishponds. They presented a lone witness (their overseer). The


Bureau of Lands argue that the lands in dispute are not contends that it is an instrumentality of the government and as
accretions. They assert that what actually happened was that the such exempted from real estate tax. That the land and buildings
respondents simply transferred their dikes simply further down of MIAA are of public dominion therefore cannot be subjected to
the river bed of the Meycauayan River. Thus, if there was any levy and auction sale.
accretion to speak of, it was man-made. Issues: Whether or not the land and buildings of MIAA are part of
the public dominion and thus cannot be the subject of levy and
auction sale.
Respondents counter that the their evidence shows that accretion
Ruling: The land and buildings of MIAA are part of the public
happened without human intervention and that the transfer of the dominion. Since the airport is devoted for public use, for the
dikes occurred only after. domestic and international travel and transportation. As part of
the public dominion the land and buildings of MIAA are outside
the commerce of man. To subject them to levy and public auction
ISSUE: is contrary to public policy. Being for public use, its ownership is
with the Republic of the Philippines
Whether accretion took place
16
Tufexis vs Olaguera
RULING:
32 Phil 654
No. Alluvion must be the exclusive work of nature. There is not
evidence that the addition to said property was made gradually
thro
Facts: During the Spanish regime, Ricardo Pardo y Cabañas was
15 allowed by the Spanish government to have the usufruct of a
public market for 40 years. Ricardo died, and the usufruct was
MANILA INTERNATIONAL AIRPORT AUTHORITY VS CA inherited by Ricardo Pardo y Pujol, his son. When Pujol became
GR No. 155650, July 20, 2006, 495 SCRA 591
indebted, his properties were sold at an auction sale, and the
Facts: usufruct was bought by Tufexis. Then a fire destroyed the market.
The Council granted Pujol the right to reconstruct the building and
MIAA failed to settle the entire amount of real estate tax due.
continue the usufruct. Tufexis complained on the ground that he
Paranaque City levied and auctioned the land and buildings of
MIAA. MIAA sought for a TRO from the CA but failed to do so had bought at the auction sale Pujol’s usufruct.
within the 60 days reglementary period, so the petition was Issue: Whether Tufexis can be given the usufruct and
dismissed. MIAA then sought for the TRO with the SC a day administration of the market
before the public auction, TRO was granted but was received by
the Paranaque City 3 hours after the public auction. MIAA



Held: Tufexis cannot be given the right because the right is of Residents complained that the closing of the canals caused
public character and could not be bought at an auction sale. For floods during the rainy season and it deprived them of their
Tufexis to take Pujol’s place would be contrary to law, for this means of transportation and fishing grounds. They demanded the
would be allowing a stranger who had not been selected by the re-opening of the canals. Subsequently, the mayor, accompanied
government, to take over a public function. On the other hand, in by the residents, went to the hacienda and opened the closed
terms of the concession given to Cabañas, were personal and dikes.
transferable only by inheritance. Tufexis, not being an heir of
Santos filed Civil Case No 448 in the CFI of Pampanga which
Cabañas, cannot therefore exercise the right.
preliminarily enjoined the mayor and others from demolishing the
dikes across the canals. The municipal officials of Macabebe
countered by filing a complaint in the same court. The CFI
rendered judgment in both cases against Santos who
17. immediately elevated the case to the Supreme Court.
Roman R. Santos v Hon. Florencio Moreno Issue:
21 SCRA 1141 Do the streams involved in this case belong to the public domain
December 4, 1967 or to the owner of Hacienda Esteban?
Facts: Held:
The Zobel famiy owned the Hacienda San Esteban. It was Pursuant to Art 71 of the Spanish Law of Waters of August 3,
devoted to the planting and cultivation of nipa palms. Accessibility 1866 and Art 408 (5) of the Spanish Civil Code, channels of
through the nipa palms deep into the hacienda posed as a creeks and brooks belong to the owners of the estates over which
problem so its administrator dug canals. By the gradual process they flow. The channels, therefore, of the streams in question,
of erosion, these canals acquired the characteristics and which may be classified as creeks, belong to the owners of
dimensions of rivers. Hacienda San Esteban. The streams, considered as canals, of
which they originally were, are of private ownership in
Later, they converted the forest of nipa groves into a web of
contemplation of Art 339 (1) of the Spanish Civil Code. Under Art
fishponds for bangus culture. They constructed dikes and closed
339, canals constructed by the State and devoted to public use
the canals criss-crossing the hacienda.
are of public ownership. Conversely, canals constructed by
Sometime in 1925 or 1926, a portion of the hacienda was sold to private persons within private lands and devoted exclusively for
Roman Santos who also transformed the swamp land into a private use must be of private ownership.
fishpond. Santos closed and built dikes across Sapang Malauling
There being a showing that the canals were constructed for the
Maragul, Quiñorang Silab, Pepangebunan, Bulacus, Nigui and
use and benefit of its owner to the exclusion of the public, the
Nasi.
canals in question are adjudged to be of private ownership.


18. ALMAGRO VS KWAN GR 175806 Facts:
Judge Taccad owned Lot 307 which is bordered by the Cagayan
River on its west. The western portion would occasionally be
Facts:
submerged in waters and reappear during the dry season. When
Respondents are the legitimate children of Kwan Chin and
Zosima Sarana, who both died intestate. Respondents filed with a survey of the land was conducted on a rainy month, a portion of
the MTC an action for recovery of possession and damages for the land that Manalo bought was then underwater and was thus
their parents’ lot. The MTC dismissed the complaint on the left unsurveyed and excluded from said Lot 307. The unsurveyed
ground that the remaining dry portion of the subject lot has portion, on the other hand, is the bed of the eastern branch of the
become foreshore land and should be returned to the public river.
domain and that plaintiff cannot use the doctrine of indefeasibility
of their Torrens title, as property in question is clearly foreshore
land. Issue: Is Manalo Entitled to the unsurvey portion of the Land by
accretion?
Issue: W/N the disputed portion of the subject lot is still private
land or has become foreshore land which forms part of the public
domain.
Ruling:
Held: The buyer did not acquire private ownership of the bed of the
The Supreme Court denied the petition and affirmed the eastern branch of the Cagayan River even if it was included in the
Resolution of the CA that the disputed land is not foreshore land. deeds of absolute sale executed by the sellers since the sellers
To qualify as foreshore land, it must be shown that the land lies
“could not have validly sold land that constituted property of
between the high and low water marks and is alternately wet and
dry according to the flow of the tide. The land's proximity to the public dominion.” The disputed property is not an accretion. It is
waters alone does not automatically make it a foreshore land. the action of the heavy rains that cause the highest ordinary level
of waters of the Cagayan River during the rainy season. The
depressed portion is a river bed and is thus considered property
19 of public domain.
G.R. NO. 92161, MARCH 18, 1991 20
(ACCRETION; RIVER: PROPERTY OF PUBLIC DOMINION)
Hilario vs. The City of Manila
G.R. No. L-19570, April 27, 1967
19. BINALAY VS MANALO Facts:
Dr. Jose Hilario was the registered owner of a large tract of land
which was inherited by Jose Hilario Jr., plaintiff. During the


lifetime of plaintiff's father, the Hilario estate was bounded on the Defendants were extracting from the public domain under proper
western side by the San Mateo River. To prevent its entry into the authorization and did not extract materials from plaintiff's
land, a bamboo and lumber post dike or ditch was constructed on property.
the northwestern side. This was further fortified by a stonewall
built on the northern side. However, in 1937, a great and
extraordinary flood occurred which inundated the entire place. 21.REPUBLIC VS. PARAÑAQUE,
The river destroyed the dike on the northwest, left its original bed
and meandered into the Hilario estate, segregating from the rest
thereof a lenticular place of land. In 1945 the U.S. Army opened a FACTS:
sand and gravel plant within the premises and started scraping,
excavating and extracting soil, gravel and sand from the nearby This is a petition for review on certiorari under Rule 45 of the
areas the River. The operations eventually extended northward 1997 Rules of Civil Procedure, on pure questions of law, assailing
into this strip of land. In 1947, the plant was turned over to
defendants who took over its operations. the January 8, 2010 Order of the Regional Trial Court, Branch
195, Paranaque City (RTC), which ruled that petitioner Philippine
Issue: Reclamation Authority (PRA) is a government owned and
Whether the new riverbanks lining the said course would be of
public ownership when a river, leaving its old bed, changes its controlled corporation (GOCC), a taxable entity, and, therefore,
original course and opens a new one through private property not exempt from payment of real property taxes.

Ruling: ISSUE: W/N Reclaimed Lands are exempt from Real Estate Tax?
All riverbanks are of public ownership including those formed
when a river leaves its old bed and opens a new course through a
private estate. RULING: The Court agrees with PRA that the subject reclaimed
lands are still part of the public domain, owned by the State and,
Art. 339 of the old Civil Code provides: therefore, exempt from payment of real estate taxes. Here, the
Property of public ownership is —
1. That devoted to public use, such as …rivers…; Since subject lands are reclaimed lands, specifically portions of the
undeniably all beds of rivers are of public ownership, it foreshore and offshore areas of Manila Bay. As such, these lands
follows that the banks, which form part of them, are also of
remain public lands and form part of the public domain.
public ownership.
Art. 372 of the old Civil Code which provides that:
Whenever a navigable or floatable river changes its course
from natural causes and opens a new bed through a
private estate, the new bed shall be of public ownership…



22. Yu Chang v. Republic, G.R. No. 171726, Feb. 23, 2011
Facts: Petitioners inherited some parcel of lots from their
deceased father and built on it buildings for commercial and
23. Laurel vs Garcia
residential use. They then proceeded to have the land registered
under the Property Registration Decree. The state opposed the 187 SCRA 797
registration as it claimed that said lots being forestland are part of
the inalienable property of the public domain. Petitioners Facts:
contended that it should no longer be considered and forest land The Roppongi property acquired from the Japanese government
as they have already built buildings for residential or commercial became the site of the Philippine Embassy until the latter was
use on the lots. transferred to Nampeidai when the Roppongi building needed
Issue: Whether or not said parcels of lots are inalienable property major repairs. President Aquino created a committee to study the
of the public domain. disposition of Philippine government properties in Tokyo and
Kobe, Japan. The President issued EO 296 entitling non-Filipino
Held: The classification of land is descriptive of its legal nature or citizens or entities to avail of separations' capital goods and
status and does not have to be descriptive of what the land services in the event of sale, lease or disposition.
actually looks like. The fact that the area within which the subject
parcels of land are located is being used for residential and
commercial purposes does not serve to convert the subject Issues: Whether or not the Chief Executive, her officers and
parcels of land into agricultural land. It is fundamental that before agents, have the authority and jurisdiction, to sell the Roppongi
any land may be declassified from the forest group and converted property.
into alienable or disposable land for agricultural purposes, there
must be a positive act from the government. Here, petitioners
failed to present well-nigh incontrovertible evidence necessary to Ruling: It is not for the President to convey valuable real property
prove their compliance of the requirements under Section 48(b) of of the government on his or her own sole will. Any such
C.A. No. 141. conveyance of real property falling under patrimonial property of
the state must be authorized and approved by a law enacted by
the Congress. It requires executive and legislative concurrence.



24
DepEd V. MARIANO TULIAO
G.R. No. 205664 June 9, 2014

FACTS: Tuliao filed an action for recovery of possession and


removal of structure with damages against the DepEd alleging
that he was the registered owner of the subject parcel of land and
that a portion of the said property was allowed by his
predecessors-in-interest to be used by the Atulayan Elementary
School (AES). Upon discovering that a structure was being
constructed on the land, he demanded that the DepED cease and
desist and vacate the property. In its defense, the DepEd denied
the material allegations and averred that it did not state a cause
of action. Even if there was, the same was already barred by
prescription and/or laches.
ISSUES: WON the action is barred by laches?
RULING/DOCTRINE:
When a landowner filed an action for recovery of
possession, and the plaintiff showed as evidence tax declarations
and a certificate of title over the property, the lone testimonial
evidence the DepEd presented is not sufficient to controvert the
landowner’s case. In addition, the landowner’s claim is not barred
by laches when the school’s possession of the property is not
adverse, and when the landowner brought suit two years after he
learned that the school is constructing a gymnasium over the
property.



In the case at bench, the heirs of Yabao merely alleged
that they are the heirs of Paciano Yabao without presenting any
25
proof why they are the latter’s heirs and in what degree or
Heirs of Yabao v. Paz Lentejas Van Der Kolk capacity.
GR No. 207266, June 25, 2014
The basis of the heirs’ claim of ownership was a mere tax
declaration that was supposedly in the name of their putative
Facts: The heirs of Yabao alleged in their complaint that they are ancestor Paciano Yabao. However, a tax declaration is not a
the sole surviving heirs of the late spouses Yabao, and that they proof of ownership; it is not a conclusive evidence of
are the absolute co-owners of the parcel of land declared in the ownership of real property. In the absence of actual, public,
name of the late Paciano Yabao. The Heirs of Yabao prayed that and adverse possession, the declaration of the land for tax
they be declared the co-owners and possessors of the subject purposes does not prove ownership. It can only be a strong
property occupied and possessed by Paz Lentejas Van Der Kolk indication of ownership if coupled with possession.
(Van Der Kolk), who also asserted claim of ownership over the
same.
Van Der Kolk started possessing the land in 1996 and refused to In the case at bench, it was Paz Lentejas who was in
vacate the same despite demands from the said heirs. Also, she possession of the property and not the heirs of Yabao.
filed a petition for original patent regarding such land. This was Consequently, the tax declaration, standing alone, is not an
timely oppsed by the heirs, according to them. acceptable proof of ownership.
Aside from these material averments in the complaint, nothing
else was presented to prove the heirs' right over the subject
property.
Issue: Whether or not the Heirs of Yabao are the rightful owners
of the subject property.
SC: No.
Ownership by the heirs cannot be established by mere lip
service and bare allegations in the complaint. As in all matters, a
party must establish his/her averments in the complaint by
sufficient evidence necessary to prove such claim.



26
Midway Maritime and Technological Foundation, petitioner v. 27. Subic Bay Legend Resorts and Casinos, Inc. vs Bernard
Marissa Castro Fernandez
GR. No. 189061 August 6,2014 September 29, 2014
G.R. No. 193426
Facts: In a mortgage over a parcel of land a contract of a 15 Facts:
year lease was entered in favor of Castro who later built a
On July 1, 1994, Bernard Fernandez, brother of Ludwig and
residential building. The mortgage was later foreclosed and the
Deoven, filed a complaint for the recovery of sum of money and
parcel of land was then sold in a public auction. After the lapsed
damages against Subic Bay Legend Resorts and Casinos, Inc
of the redemption period, the bank then sold it to Cloma. Cloma
(SBL). According to him, he went to the casino on June 13, 1997
leased the property with herein petitioner which also leased the
and handed to his brothers $6,000.00 worth of chips belonging to
residential building owned by the Castro with payment of monthly
him, for use at the casino. Thereat, the company personnel
rentals. Cloma then sold the property to her daughter which is the
accosted his brothers and confiscated his casino chips worth
wife of the President of herein petitioner. Petitioner now files an
$5,900.00. The company refused to return the same despite their
action for ownership and recovery of possession.
demand. Ludwig and Deoven were accused of stealing casino
Issue: Whether or not there was a lease agreement as regards to chips from SBL. They were made to confess that the chips were
the residential building? supplied by a casino employee, Michael Cabrera.
Held: Yes, it is settled that once a contract of lease is shown to Issue: Whether or not Bernard is the lawful possessor of the
exist between the parties, the lessee cannot by any proof, casino chips, entitling him to collect from the casino and award
however strong, overturn the conclusive presumption that the damages.
lessor has a better right of possession to the subject premises.
Ruling:
The payment of rentals confirms the existence of the lease
agreement. No. There is no basis to suppose that the casino chips found in
Ludwig’s and Deoven’s possession were stolen. Unless the
independent fact that Cabrera stole the chips can be proved, it
cannot be said that the chips must be confiscated when found to
be in the brothers’ possession.
Though casino chips do not constitute legal tender, there is no
law which prohibits their use or trade outside of the casino which
issues them. Since casino chips are considered to have been



exchanged with their corresponding representative value, it is courts may pass upon that issue but only to determine who
with more reason that the Court should require SBL to prove that between the parties has the better right to possess the property.
the chips confiscated were indeed stolen from them. If SBL 2.) As between the petitioner's Deed of Absolute Sale and
cannot prove its loss, then Art. 559 cannot apply, and the the respondents' TCT, the latter must prevail. A title issued under
presumption that the chips were exchanged for value remains. the Torrens system is entitled to all the attributes of property
ownership, which necessarily includes possession. Hence, as
Case 28: G.R. No. 199448, 12 November 2014
holders of the Torrens title over the subject land, the respondents
ROLANDO S. ABADILLA, JR., Petitioner, v. SPOUSES
are entitled to its possession.
BONIFACIO P. OBRERO AND BERNABELA N.
OBRERO, Respondents.
29
FACTS: Respondent Spouses claimed that they are the HEIRS OF EUGENIO LOPEZ VS QUERUBIN
registered owners of the land in question by virtue of the TCT
Facts:
issued under their name. They claimed that they were in
possession thereof based on improvements erected therein In the Deed of Absolute Sale, the vendors-applicants obligated
utilized for residential and business purposes prior to the alleged themselves to file in the land registration case the necessary
acts of petitioner who forcibly fenced the perimeter of the land motion or manifestation in order that the certificates of title will be
with barbed wire. Petitioner, on the other hand, claimed that the issued in the name of Eugenio Lopez or his successors-in-
land was sold by the spouses to his late father as evidenced by a interest. This was not complied with, the Lopez heirs prayed that
Deed of Absolute Sale. Being one of the heirs, he is one of the the decrees of registration and original certificate of titles be
owners thereof. Despite the sale, respondents supposedly declared null and void, and the subject properties be issued in
attempted to remove the fence and even built concrete structures their names.
on the land using it for dwelling purposes.

ISSUE/s: 1.) Whether or not an ejectment proceeding is proper in Issues: W/N Contention of Lopez Heirs is correct
the case at bar.
2.) Whether or not respondents are entitled to land’s
possession. Ruling:

HELD: 1.) Yes. Ejectment proceedings are summary Nowhere can it be inferred in Section 22 of PD No. 1529 that the
proceedings. Issues as to ownership are not involved in the buyer of the property automatically becomes a party to the land
action; evidence thereon is not admissible, except only for the registration case after complying with the requirements of the
purpose of determining the issue of possession. Thus, where the aforesaid provision of law. To rectify their situation, the Lopez
parties to an ejectment case raise the issue of ownership, the heirs could have resorted to Section 108 of PD No. 1529 in order
to correct the palpable mistakes in the certificates of title in this


case. Thus, this case is remanded to the land registration court No. Article 434 of the Civil Code provides that to successfully
for further proceedings, subject to the Lopez heirs' compliance maintain an action to recover the ownership of a real property, the
with the requisites of Section 108 of Presidential Decree No. person who claims a better right to it must prove two (2) things:
1529. first, the identity of the land claimed; and second, his title thereto.

In this case, petitioners failed to prove the identity of the parcels


of land sought to be recovered and their title thereto as they failed
to prove that Lot Nos. 1625 and 1626 were part of or used to be
part of Lot No. 1197.

30
GR No. 152141 August 8, 2011
31
DEL FIERRO vs SEGUIRAN
Del Rosario v. Gerry Roxas Foundation, Inc. G.R. No. 170575.
Facts:
June 8, 2011. First Division; Del Castillo, J.
Petitioners contend that they are the owners of Lot Nos. 1625 and
1626 as the Municipal Court declared them to be the ones in
possession of the property in an ejectment case against Facts: del Rosario appears to be the registered owner of a lot
respondents, which decision was affirmed on appeal. However, covered by a TCT located in Roxas CIty. Gerry Roxas
as stated by the trial court and the Court of Appeals, the property Foundation, Inc., (GRFI) took possession and occupancy of said
involved in the ejectment case was Lot No. 1197. It was never land, in the character of a lessee, by virtue of a MOA it entered
mentioned in the respective decisions of the Municipal Court and with Roxas City. Later, Spouses del Rosario served notices upon
the CFI of that the portion intruded upon was Lot Nos. 1625 and the GRFI to vacate the premises of said land. GRFI, however, did
1626. not heed such notices because it still has the legal right to
continue its possession and occupancy of said land.
Issue: Whether or not there is a cause of action for forcible entry
Issue: whether petitioners are entitled to reconveyance of Lot
and not an unlawful detainer?
Nos. 1625 and 1626,
Held: Yes. In forcible entry, one is deprived of physical
possession of any land or building by means of force,
Ruling: intimidation, threat, strategy, or stealth. Where the defendant’s


possession of the property is illegal ab initio, the summary action 33
for forcible entry (detentacion) is the remedy to recover
Bienvenido Barrientos, petitioner
possession. In their Complaint, the spouses maintained that the
-versus-
GRFI took possession and control of the subject property without
Mario Rapal, respondent
any contractual or legal basis. Thus, it is one of forcible entry, or
the forcible exclusion of the original possessor by a person who
G.R. No. 169594 20 July 2011
has entered without right, and not unlawful detainer.
FACTS:
Rapal acquired a parcel of land from Nataviovia a notarized Deed
of Transfer of Possessory Right and thereafter built a semi-
concrete house. Respondent allowed Barrientos and his family to
stay on the subject land as caretakers on the condition that the
latter shall vacate the premises when the former needs the
property. However, when respondent eventually demanded on
several occasions that the petitioner leave the property, the latter
refused. Rapal was prompted to file a case for Unlawful Detainer
against Barrientos.

32. Jose vs. Alfuerto, G.R. No. 169380 ISSUE: Whether petitioner has the better right over the
FACTS: Chua Sing leased a property to petitioner. Respondents subject property.
already occupied the property even before the lease contract was
executed. When Chua Sing and petitioner signed the lease HELD:
contract, the petitioner demanded in writing that the respondents No. The respondent clearly makes out a case for unlawful
vacate the property within 30 days. detainer, since petitioner's occupation of the subject property was
by mere tolerance. A person who occupies the land of another at
ISSUE: Whether or not this is a case for unlawful detainer and not the latter's tolerance or permission, without any contract between
for recovery of possession. them, is necessarily bound by an implied promise that he will
vacate the same upon demand, failing which a summary action
RULING: No. Unlawful detainer is not the proper remedy.
for ejectment is the proper remedy against them.
Respondent’s occupancy was unlawful from the start and was
bereft of contractual or legal basis. If the possession was unlawful
from the start, an action for unlawful detainer would not be the
proper remedy and should be dismissed.



34 raised in a forcible entry case. It must be stated that regardless
of the actual condition of title to the property, the party in a
GERMAN MANAGEMENT & SERVICES, INC. V COURT OF
peaceable quiet possession shall not be turned out by a strong
APPEALS
hand, violence or terror. Thus, a party who can prove prior
177 SCRA 495 (1989)
possession can recover such possession even against the owner
himself. Whatever may be the character of his prior possession,
FACTS:
if he has in favor priority in time, he has security that entitles him
Spouses Jose issued a power of attorney in favor of petitio to remain on the property until he has been lawfully ejected by a
ner for the development of their parcel of land into a person having a better right by accion publiciana or accion
subdivision. Private respondents reivindicatoria.
were occupying the land and petitioner advised them to vac
ate but they refused. Thereafter, petitioner continued their
development and 36
construction. Respondents then filed a case for forcible entry.
NPC v. Ibrahim
The trial court dismissed the complaint and this was reversed by
the CA. FACTS: NPC constructed underground tunnels on the property of
the respondent without their knowledge, consent, and any
ISSUE: expropriation proceeding. NPC contented that it constructed an
easement on the property.
Whether or not private respondents are entitled to file a forcible
entry case against petitioner?
ISSUE: Whether or not the owners of the property are entitled to
HELD:
just compensation.
Notwithstanding petitioner’s claim that it was duly authorized
by the
owners to develop the subject property, private respondents RULING: Yes. Despite the fact that NPC only occupied the sub-
as actual possessors, can commence a forcible entry case terrain portion, it is liable to pay not only an easement fee but
against petitioner because ownership is not in issue. Forcible rather the full compensation for the land because the nature of
entry is merely a quieting process, and never determines the the easement practically deprived the owners of its normal
actual title to an estate. Title is not involved. beneficial use.

Although admittedly petitioner may validly claim ownership b 37


ased on the muniment of title it presented, such evidence PALERO-TAN vs. URDANETA
doesn’t responsively address the issue of prior actual possession


FACTS: Urdaneta found jewelry inside the RTC office. Tan is the condominium building located Cubao, Quezon City. For failure of
owner of the jewelry. Urdaneta did not return the jewelry instead Villasi to fully pay the contract price despite several demands,
he threw it away when his wife suspected that we bought the FGCI initiated a suit for collection of sum of money. Villasi filed an
jewelry for a mistress. However, it was found out that he knew answer specifically denying the material allegations of the
that Tan had lost the jewelry and that he did not bother to inform complaint. Contending that FGCI has no cause of action against
her. her, Villasi averred that she delivered the total amount of P
7,490,325.10 to FGCI but the latter accomplished only 28% of the
project. To enforce her right as prevailing party, Villasi filed a
Motion for Execution. To satisfy the judgment, the sheriff levied
ISSUE: WON Urdaneta can be faulted for throwing away the on a building located Kalayaan Avenue, Quezon City. (While the
jewelry he found. building) which was declared for taxation purposes in the name of
FGCI while the lots in which it was erected were registered in the
names of the Spouses Garcia. The Spouses Garcia argued that
HELD: Yes, Urdaneta can be faulted for throwing away the the building covered by the levy was mistakenly assessed by the
jewelry he found. When a person who finds a thing that has been City Assessor in the name of FGCI and that it could not be levied
lost or mislaid by the owner takes the thing into his hands, he upon not being owned by the judgment debtor.
acquires physical custody only and does not become vested with
legal possession. In assuming such custody, the finder is charged The spouses Garcia filed an Affidavit of Third Party Claim and a
Motion to Set Aside Notice of Sale on Execution, claiming that
with the obligation of restoring the thing to its owner. It is thus
they are the lawful owners of the property.
respondent’s duty to report to his superior or his officemates that
he found something. The RTC hereby orders Deputy Sheriff Angel Doroni to suspend
38 or hold in abeyance the conduct of the sale on execution of the
buildings levied upon by him, until further orders from the Court.
G.R. No. 190106
January 15, 2014 Villasi timely filed a Petition for Certiorari before the CA. The
petition was dismissed.
MAGDALENA T. VILLASI v. FILOMENO GARCIA substituted by
his heirs, namely, ERMELINDA H. GARCIA, LIZA GARCIA- Villasi filed a petition for review on certiorari pursuant to Rule 45
GONZALEZ, THERESA GARCIA-TIANGSON, MARIVIC H. of the Revised Rules of Court, assailing the decision rendered by
GARCIA, MARLENE GARCIA-MOMIN, GERARDO H. GARCIA, the CA.
GIDEON H. GARCIA and GENEROSO H. GARCIA, and
ERMELINDA H. GARCIA Issue:
WON the general rule on accession can be applied in the
Facts: case at bar.
Villasi engaged the services of respondent Fil-Garcia Held:
Construction, Inc. (FGCI) to construct a seven-storey


No. While it is a horn-book doctrine that the accessory follows the
principal, that is, the ownership of the property gives the right by
accession to everything which is produced thereby, or which is
incorporated or attached thereto, either naturally or artificially,
such rule is not without exception. In cases where there is a clear
and convincing evidence to prove that the principal and the
accessory are not owned by one and the same person or entity,
the presumption shall not be applied and the actual ownership
shall be upheld.

When there are factual and evidentiary proof that the building and
the lot on which it stands are owned by different persons, they
shall be treated separately. As such, the building or the lot, as the
case may be, can be made liable to answer for the obligation of
its respective owner.

The SC granted the petition. The assailed Decision and


Resolution of the Court of Appeals are hereby REVERSED and
SET ASIDE. The Deputy Sheriff is hereby directed to proceed
with the conduct of the sale on execution of the levied building.



39 HLURB, the City Building Official and the RD in Quezon City, of
the illegal constructions being made thereon. The HLURB issued
BPI vs Sanchez
a Cease and Decease Order enjoining Garcia / TSEI from further
Case Digest GR 179518 Nov 19 2014
developing and selling the townhouses. Such orders were left
unheeded. In fact, Garcia were already able to sell many of the
Facts:
units to different individuals and entities, and even mortgaged the
The Sanchezes entered into an agreement with Garcia (doing
property. Consequently, the Sanchezes filed before the RTC a
business in the name of TSEI) to sell for P 1.850 million their
complaint for rescission, restitution and damages with TRO.
parcel of land, with an earnest money of 50k. They agreed that
Garcia shall pay the purchase price in cash once the property is
The purchasers and mortgagee who are the intervenors in this
vacated. The Sanchezes entrusted to Garcia the owner’s copy of
case were found by the court to be in bad faith. On the other
TCT because it was agreed that he shall take care of all the
hand, the Sanchezes were held to be in good faith and not
documentations necessary for the transaction.
negligent.
Immediately after the property was vacated, Garcia took
Issue 1: W/N rescission of the contract was barred by the
possession and began constructing townhouses thereon without
subsequent transfer of the property
the Sanchezes’ knowledge and consent. While these
No. Under Article 1191 of the Civil Code, rescission is available to
developments were ongoing, Garcia failed to pay the purchase
a party in a reciprocal obligation where one party fails to comply
price. Subsequently, the Sanchezes were given six checks
with it. As an exception to this rule, Article 1385 provides that
representing the amount of the purchase price. Four of these
rescission shall not take place if the subject matter of the prior
checks were postdated, thus further delaying their overdue
agreement is already in the hands of a third party who did not act
payment. To properly document the check payments, they made
in bad faith.
an agreement stipulating that if one of the checks were
Here, the failure of Garcia/TSEI to pay the consideration for the
dishonored, the Sanchezes may rescind the contract.
sale of the property entitled the Sanchezes to rescind the
Agreement. And in view of the finding that the intervenors acted
The last two checks were dishonored, so the Sanchezes
in bad faith in purchasing the property from Garcia, the
rescinded the contract and demanded from Garcia the return of
subsequent transfer in their favor did not and cannot bar
the TCT. However, Garcia refused to return the documents and
rescission.
vacate the property.

Issue 2: W/N Article 449 – 450 of the Civil Code is applicable in


Meanwhile, the Sanchezes found out that Garcia/TSEI were
the case at bar.
selling townhouses situated in the property. So they informed the


Yes. Bad faith on the part of the purchasers leads to the 40. FLOREZA VS. EVANGELISTA, 96 SCRA 130, February 21,
application of Art 449-450. 1980
Consequently, the Sanchezes have the following options: (1)
acquire the property with the townhouses and other buildings and Facts:
improvements that may be thereon without indemnifying TSEI or The Evangelistas were the owner of a residential lot in
the intervenors; (2) demand from TSEI or the intervenors to Rizal with an area of 204.08 sq.m. assessed at P410. They
demolish what has been built on the property at the expense of borrowed P100 from Floreza. Floreza occupied the residential lot
and built a house of light material (barong-barong) with the
TSEI or the intervenors; or (3) ask the intervenors to pay the price consent of the Evangelistas. Additional loans were made by the
of the land. Evangelistas.

As such, the Sanchezes must choose from among these options Floreza demolished the house of light material and
within 30 days from finality of the decision. Should the Sanchezes constructed a house of strong material. Floreza has not been
opt to ask from the intervenors the value of the land, the case paying any rentals since the beginning of their transactions.
Eventually, Evangelistas sold, with a right to repurchase within 6
shall be remanded to the RTC for the sole purpose of years, their land to Floreza. 7 months before the expiry of the
determining the fair market value of the lot at the time the same repurchase period, the evangelistas were able to pay in full.
were taken from the Sanchezes. If the Sanchezes decide to Floreza refused to vacate the lot unless he was first reimbursed
appropriate the townhouses, other structures and improvements for the value of the house he built.
as their own pursuant to Art 449, then the intervenors-purchasers
Issue:
shall be ordered to vacate said premises within a reasonable time 1. W/N Floreza was entitled to reimbursement of the cost
from notice of the finality of the decision by the Sanchezes. They of his house.
have a right to recover their investment in the townhouses from 2. W/N his heirs who replaced Floreza should pay rental
Garcia and TSEI. If the Sanchezes do not want to make use of of the land.
the townhouses and improvements on the subject lot, then the
HELD:
purchasers can be ordered to demolish said townhouses or if 1. No. Issue of reimbursement is not moot because if Floreza
they don’t demolish the same within a reasonable time, then it has no right of retention, then he must pay damages in the
can be demolished at their expense. On the 3rd option, if the form of rentals. Art. 448 is inapplicable because it applies only
Sanchezes do not want to appropriate the townhouses or when the builder is in good faith where he believed that he
have the same demolished, then they can ask that the townhouse had a right to build. Art. 453 is also not applicable because it
requires both of the parties to be in bad faith. Neither Art.
purchasers pay to them the fair market value of the respective
1616 applicable because Floreza is not a vendee a retro. The
areas allotted to their respective townhouses subject of their house was already constructed in 1945 in light materials, even
deeds of sale. before the pacto de retro was entered into in 1949. Floreza


cannot be classified as a builder in good faith nor a vendee a The members of the Bulaong Group sued. They filed
retro, who made useful imporvements during the pacto de several individual complaints with the Court of First Instance
retro, he has no right to reimbursement of the value of the seeking recovery of their stalls from the Mercado Group as well
house, much less to the retention of the premisers until he is as damages. Their theory was anchored on their claimed
paid. He may, however, remove such improvements shoul it ownership of the stalls constructed by them at their own expense,
be possible to do so without damage to the property. and their resulting right, as such owners, to sub-lease the stalls,
2. Yes. From the time the redemption price was paid in January and necessarily, to recover them from any person withholding
3, 1955, Floreza’s right to use the residential lot without rent possession thereof from them.
ceased. He should be held liable for damages in the form of
rentals for the continued use of the lot for P10 monthly from On October 24,1975, respondent Judge rendered a
January 3, 1995 until the house was removed and the summary judgment in all the cases. It rejected the claim of the
property vacated by Floreza or his heir. Municipality of Baliuag that it had automatically acquired
ownership of the new stalls constructed after the old stalls had
# 41 been razed by fire, declaring the members of the Bulaong Group
Mercado v. CA to be builders in good faith, entitled to retain possession of the
G.R. No. L-44001. June 10, 1988. stalls respectively put up by them until and unless indemnified for
the value thereof. The decision also declared that the Bulaong
TOPIC: Builder in Good Faith and Mercado Groups had executed the subleasing agreements
FACTS: with full awareness that they were thereby violating Ordinance
No. 14; they were thus in pari delicto, and hence had no cause of
The private respondents Bulaong Group, had for many action one against the other and no right to recover whatever had
years been individual lessees of stalls in the public market of been given or demand performance of anything undertaken. The
Baliuag, Bulacan; from 1956 to 1972. The market was destroyed judgment therefore decreed (1) the annulment of the leases
by fire on February 17, 1956; the members of the Bulaong Group between the Municipality and the individuals comprising the
constructed new stalls therein at their expense; and they Mercado Group (the defendants who had taken over the original
thereafter paid rentals thereon to the Municipality of Baliuag. leases of the Bulaong Group); and (2) the payment to the
individual members of the Bulaong Group (the plaintiffs) of the
In 1972, the members of the group sub-leased their stated, adjudicated value of the stalls, with interest.
individual stalls to other persons, referred to as the Mercado
Group. After the Mercado Group had been in possession of the The members of the Mercado Group are now before this
market stalls for some months, as sub-lessees of the Bulaong Court on an appeal by certiorari, this time timely taken, assailing
Group, the municipal officials of Baliuag cancelled the long the above rulings of the Court of Appeals.
standing leases of the Bulaong Group and declared the persons
comprising the Mercado Group as the rightful lessees of the stalls ISSUE: WON the Bulaong group is a builder in good faith.
in question, in substitution of the former.
HELD:



NO. It is not a builder in good faith. It was held that “to be construction of a house were made on said lot by the latter. The
deemed a builder in good faith, it is essential that a person assert lot was then sold to petitioner Rosendo Balucanag, who, through
title to the land on which he builds; i.e., that he be a possessor in counsel, demanded Stohner to vacate the premises due to failure
concept of owner, and that he be unaware ‘that there exists in his to pay rent. Stohner, also through counsel, claimed that he was a
title or mode of acquisition any flaw which invalidates it.’ builder in good faith of the residential house constructed on the
lot.
Lessees cannot be considered builders in good faith
(taken from Haystacks, by Berne Guerrero) The City Court of Manila rendered judgment on the ejectment suit
in favor of petitioner. However, Judge Fransisco, who presided
The members of the Bulaong group were admittedly over the Court of First Instance of Manila where the appeal was
lessees of space in the public market; they therefore could not, made, set the judgment aside saying that Stohner was indeed a
and in truth never did make the claim, that they were owners of builder in good faith, invoking Art. 448 and 546. Thus, Stohner
any part of the land occupied by the market so that in respect of
cannot be ejected from the lot without being reimbursed for the
any new structure put up by them thereon, they could be deemed
value of the improvements.
builders in good faith (in accordance with Article 526 of the Civil
Code). To be deemed a builder in good faith, it is essential that a ISSUE:
person assert title to the land on which he builds; i.e., that he be a
possessor in concept of owner, and that he be unaware “that W/N Stohner is a builder in good faith
there exists in his title or mode of acquisition any flaw which
HELD:
invalidates it. It is such a builder in good faith who is given the
right to retain the thing, even as against the real owner, until he No. The Court stated that Art. 448, relied upon by respondent
has been reimbursed in full not only for the necessary expenses judge, applies only to a case where one builds on land in the
but also for useful expenses. On the other hand, unlike the belief that he is the owner thereof and it does not apply where
builder in good faith, a lessee who “makes in good faith useful one’s only interest in the land is that of a lessee under a rental
improvements which are suitable to the use for which the lease is
contract.
intended, without altering the form or substance of the property
leased,” can only claim payment of “one-half of the value of the Also, as was decided by the Court in Lopez, Inc. v Phil. And
improvements” or, “should the lessor refuse to reimburse said Eastern Trading Co., Inc., a lessee cannot be considered a
amount, remove the improvements, even though the principal possessor in good faith since he knows beforehand that he is not
thing may suffer damage thereby.” the owner of the land, and cannot deny the ownership or title of
his lessor.
042 Balucanag v Fransisco [GR L-33422]
NOTE:
FACTS:
#You can choose not to include the statements that follow.
Mrs. Charvet leased to Richard Stohner a 177.50 square meter
lot, wherein improvements consisting of fillings on the land and



The Court also stated that the law applicable to the case at bar CASE DIGEST 43
would be Art. 1687.
[G.R. No. 149295. September 23, 2003]
PHILIPPINE NATIONAL BANK, petitioner, vs. GENEROSO DE
JESUS

FACTS:

Through verification survey of the property, Respondent


discovered that the northern portion of the lot was being
encroached upon by a building of petitioner to the extent of 124
square meters. Despite two letters of demand sent by
respondent, petitioner failed and refused to vacate the area.
Petitioner asserted that when it acquired the lot and the building
from Mayor Ignacio, the then owner, encroachment already was
in existence and Ignacio offered to sell the area in question to
petitioner. The sale, however, did not materialize when, without
the knowledge and consent of petitioner, Mayor Ignacio later
mortgaged the lot to the DBP.

ISSUE: W/N PETITIONER IS A BUILDER IN GOOD FAITH

HELD:

Applied to possession, one is considered in good faith if he is not


aware that there exists in his title or mode of acquisition any flaw
which invalidates it. Evidently, petitioner was quite aware, and
indeed advised, prior to its acquisition of the land and building
from Ignacio that a part of the building sold to it stood on the land
not covered by the land conveyed to it. Equally significant is the
fact that the building, constructed on the land by Ignacio, has in
actuality been part of the property transferred to petitioner. Under
Art 448, where the true owner himself is the builder of works on
his own land, the issue of good faith or bad faith is entirely
irrelevant.

– #44: Javier vs Concepcion 94 scra 212


(petition for review by certiorari) ARTICLE 546 of Civil Code
FACTS
Respondents allege that Lot 12 is part of Lot 6 covered by a TCT MADRONO – 45
under their name. In their Answer, petitioners denied such claim Nuguid v. CA G.R. 151815
and pointed out that Lot 6 could not include Lot 12 because there
exists a big river more than 50 meters wide and more than 20
meters deep between the two lots. Respondents filed an action FACTS: Pedro P. Pecson owned a commercial lot on which he
for reconveyance. CA held that respondents are the owners of built a four-door two-storey apartment building. For failure to pay
Lot 6 and Javier did not act in bad faith in occupying the land.
realty taxes, the lot was sold at public auction by the City
Treasurer of Quezon City to Mamerto Nepomuceno, who in turn
ISSUES sold it to the spouses Juan and Erlinda Nuguid. However, the
w/n respondents should pay for the improvements introduced by RTC ruled that the four-door two-storey apartment building was
petitioners? not included in the auction sale. Petitioners contend that their
failure to pay the full price for the improvements will, at most,
Held entitle respondent to be restored to possession, but not to collect
any rentals.
As possessors in good faith, petitioners are entitled to the fruits
received before their possession was legally interrupted upon
receipt of judicial summons in connection with the filing of the ISSUE: WON THE PETITIONERS ARE LIABLE TO PAY RENT
complaint for reconveyance. Petitioners should also be refunded OVER AND ABOVE THE CURRENT MARKET VALUE OF THE
the necessary and useful expenses, with the right to retain the
IMPROVEMENT.
land until reimbursed of the same, pursuant to Article 546 of the
Civil Code. Under the said provision, respondents have the option
to refund the amount of useful expenses or to pay the increase in
value which the land may have acquired by reason thereof. HELD: YES. Since petitioners opted to appropriate the
improvement for themselves when they applied for a writ of
execution despite knowledge that the auction sale did not include
the apartment building, they could not benefit from the lot’s
improvement, until they reimbursed the improver in full, based on
the current market value of the property. Accordingly, a builder in
good faith (Pecson) cannot be compelled to pay rentals during



the period of retention nor be disturbed in his possession by 46. Ballatan vs CA
ordering him to vacate.
FACTS:
When Ballatan constructed her house in her lot, she noticed that
the concrete fence and side pathway of the adjoining house of
respondent Winston Go encroached on the entire length of the
eastern side of her property. Ballatan made written demands to
the respondent to dismantle and move their improvements.

ISSUE: What are the remedies available to petitioner Ballatan,


the landowner?

HELD:
Pursuant to Art. 448, Ballatan has two options. Firstly, the land
owner may buy the improvements and the 2nd option is to oblige
the builders to buy the land given that the value of the land is not
considerably more than the buildings or tree; otherwise
respondents may be compelled to pay rent.

In the event the landowner elects to sell the land to the builder in
good faith, the price must be fixed at the prevailing market value
at the time of payment. In the event of the failure of the builder to
pay the land, after the owner thereof has chosen this alternative,
the builder’s right of retention provided in Article 546 is also lost.

47
HEIRS OF EMILIANO NAVARRO
48. VDA DE NAZARENO V CA
vs.
INTERMEDIATE APPELLATE COURT AND HEIRS OF
Facts : Mr. Salasan and Mr. Rabaya leased lots owned by
SINFOROSO PASCUAL [G.R. No. 68166. February 12, 1997]
Petitioner Nazareno. Such lots were formed due to accretion.
FACTS: The applicant owns the property immediately adjoining Salasan allegedly stopped paying rentals hence Petitioners filed a
the land sought to be registered. Between the Talisay River and case for ejectment. Before the death of Nazareno he caused the
the Bulacan River is the property of applicants with both rivers approval by the Bureau of Lands of the survey plan with a view of
acting as the boundary to said land. The subject land was formed perfecting his title over the accretion being which Salasan and
at the tip or apex of appellants' land adjacent to the north in front Rabuya were ejected from. It was protested by Salasan. Upon
of Manila Bay adding thereto the land now sought to be investigation, it was recommended that the survey plan be
registered. cancelled and Nazareno was ordered to vacate the portion of
land adjudicated to him and private respondent Salasan be
placed in possession thereof.
ISSUE: Whether or not the subject land deemed an accretion
formed by the action of the two rivers that naturally accrues in
favor of the riparian owner. ISSUE: WON the subject land is public land.

RULING: Riparian owners are, strictly speaking, distinct from RULING: Since the subject land was the direct result of the
owners, the latter being owners of lands bordering the shore of dumping of sawdust by the Sun Valley Lumber Co., the accretion
the sea or lakes or other tidal waters. was man-made, hence, Art. 457 does not apply. Ergo, the subject
land is part of the public domain.

49. G.R. NO. L-3788 DECEMBER 21, 1907 PEDRO P. ROXAS,


PETITIONER -APPELLEE , VS . JULIA TUASON, THE



MUNICIPALITY OF SAN PEDRO MACATI, AND ALEJANDRO taxation purposes in the name of Francisco Gerardo. The
AND CONSOLACION AGUIRRE, RESPONDENTS-APPELLANTS . respondents then sold the land to Apostol and executed an extra-
judicial partition. During the time such was executed, there were
already signs of accretion of 3 hectares. The accretion and the
FACTS: Roxas applied for the registration of the estate, the land was declared by Domingo for taxation purposes under his
Hacienda de San Pedro Macati. The owners of the adjoining name. On the other hand, petitioners contend that their
properties, one of them, Julia Tuason, set forth her opposition for predecessor, Carniyan, was the owner of the accretion of the
the reason that two old monuments which had separated their subject land. Carniyan acquired the land from his father-in-law, as
properties had been pulled down and new ones erected without evidenced by an Absolute Deed of Sale. Antonio declared the
her consent, and in her opinion the latter included a considerable land for taxation purposes and even had an OCT pursuant to
portion of the land owned by her. The petitioner affirms that the Free Patent, which includes the accretion portion and the land
real boundary of the hacienda on the side that adjoins the land of
Tuason was and still is a creek separating both properties, and
that in former years said creek was wider that at the present time. ISSUE: WONApostol and the Gerardo’s have better title over the
accretion than Cureg
Held: NO. The tax declarations presented are not sufficient
ISSUE: WON the accretion is included in the boundary line of the
evidence to prove ownership. Onthe other hand, petitioners have
parties' contiguous premises.
an OCT under the name of Antonio Carniyan to show.
Taxdeclarations are not conclusive evidence of ownership and
should not prevail over a decree ofregistration. A decree of
RULING: The right of the owners of the bank adjacent to rivers to registration bars all claims and rights which arose or may have
the accretion which they receive by virtue of the action of the existedprior the decree. The issuance of the decree quiets the
waters of the river is ipso jure and there is no need of an action of land.
the owner of the bank to possess the new addition since it
belongs to him by the very fact of the addition. 51
50 Agne v. Director of Lands
Cureg vs IAC 177 SCRA 313 (1989) 181 SCRA 793, 805 (1990)
Facts: Facts: Private respondents filed in for recovery of possesion of
Respondents, except Apostol, are the heirs of Domingo Gerardo, land stating that they are the real owners of a land in possession
the owner of the subject land. Domingo’s father, Francisco, and of the petitioners. The latter answered that the said land was
predecessors-in-interest have been in possession of the land, formerly part of a river owned by them by reason of accretion due
adverse to all other claimants. Such land was declared for


to a big flood in 1920; and since then, they had occupied it openly ISSUE: WON the petitioners have title to the property that would
and adversely as riparian owners. support an action for quieting of title.

Issue: WON the petitioners owned the abandoned river bed. RULING: No. The Office of the President held that such lands
belong to the public domain and are no longer registrable under
the Land Registration Act. In an action for quieting of title the two
Held: Yes. The petitioer own the abandoned riverbed. There need indispensable requisites in an action to quiet title are: (1) that the
be no act on their part to subject the old river bed to their plaintiff has a legal or equitable title to or interest in the real
ownership, as it is subject thereto ipso jure from the moment the property subject of the action; and (2) that there is a cloud on his
mode of acquisition becomes evident, without need of any formal title by reason of any instrument, record, deed, claim,
act of acquisition. Such abandoned riverbed had fallen to the encumbrance or proceeding, which must be shown to be in fact
private ownership of the owner of the land through which the new invalid or inoperative.
river bed passes even without any formal act of his will and any
unauthorized occupant thereof will be considered as a trespasser.
– 53
– 52
CAMPIT v GRIPA, et. al.
HEIRS OF PACIFICO POCDO vs AVILA G.R. No.199146 GR No. 195443
MARCH 19,2014 September 17, 2014

FACTS: Pacifico Pocdo, who was later substituted by his heirs


upon his death, filed a complaint to quiet title over a 1,728–
square meter property (disputed property) located in Camp 7, FACTS: Respondents Gripa, et. al. occupied a 2.7 hectare
Baguio City. The heirs filed their ancestral land claims with the agricultural land situated in Umangan, Mangatarem, Pangasinan
DENR thus Certificates of Ancestral Lands Claims (CALS) were covered by TCT No. 122237 issued in the petitioner’s name. The
issued. However, the DENR had already declared the disputed petitioner claimed to have purchased the property from his father
property as public land. Thus, the claim of petitioners to quiet title Jose Campit in the year 1977. On the other hand, respondents
is not proper since they do not have title over the disputed claimed to be the rightful owners of the subject property, as
property. The Court of Appeals ruled that petitioners, in raising earlier adjudged by the court in Civil Case No. 11858 decided on
the issue of quieting of title, failed to allege any legal or equitable June 12, 1961, and in Civil Case No. 15357 decided on August 8,
title to quiet. 1978. The Court, in these cases, cancelled the titles of the
petitioner and his father Jose because they were obtained
through the misrepresentation of the petitioner’s grandfather,



Isidro Campit. The respondents have long intended to divide the of respondent’s failure to secure a final deed of sale and/or
properties among themselves but despite formal demands, the conveyance 13 years after registration of the Sheriff’s Certificate
petitioner refused to surrender the said land. The respondents of Title.
filed a new case for annulment and cancellation of title with the
ISSUE:
RTC on Aug. 15, 2003 and ruled in favor of respondents. The CA
affirmed the RTC’s decision. WON the complaint for quiet title will prosper
HELD:
ISSUE: Whether or not petitioner is the rightful owner of the No. All rights, title, interest, and claim of the spouses De Guzman
subject property? to the subject property was already acquired by respondent upon
the expiration of the one-year redemption period without
redemption being made. The execution of the final deed of sale
HELD: No. Petitioner is not the true owner of the subject and/or conveyance to respondent is a mere formality and
agricultural land. The issue on the validity of the petitioner’s title confirmation of the title already vested in respondent. The
to the subject property has long been settled in Civil Case No. right acquired by the purchaser at an execution sale is inchoate
15357, where the court, in its decision dated August 8, 1978, and does not become absolute until after the expiration of the
which became final and executory on July 19, 1979, had found redemption period without the right of redemption having been
and declared the petitioner’s title null and void by reason of fraud exercised. Since the spouses failed to redeem the subject
and misrepresentation. The Torrens system of registration cannot property within the period allowed by law, they have been
be used to protect a usurper from the true owner, nor can it be divested of their rights over the property.
used as a shield for the commission of fraud, or to permit one to
enrich oneself at the expense of others.
54
DE GUZMAN VS. TABANGAO REALTY INC. (2015)
FACTS: Herminio De Guzman and his siblings filed a complaint
for quiet title against respondent Filipinas Shell Petroleum
Corporation (FSPC) over the parcel of land they inherited from
their parents. The subject property had long been levied upon
and sold to respondent at an execution sale. The only remaining
right of petitioners’ predecessors-in-interest over the property was
the right to redeem. Petitioners’ predecessors-in-interest failed to
redeem the property. Petitioners claimed that the acquisition of
the subject property at the execution sale was not valid because


55 late Maria De La Concepcion Vidal in Lot 26 and other lots of the
Maysilo Estate was subject to the condition, that something
FIRST DIVISION
remained of said lot in the intestate estate of said deceased that
G.R. No. 160728
have not been disposed of. Therefore, the subsequent
conveyances to E. Hipolito and then to the peititoner were still
CLT REALTY DEVELOPMENT CORPORATION, Petitioner,
subject to such condition.
vs.
PHIL-VILLE DEVELOPMENT AND HOUSING CORPORATION, 56. Chung vs. Mondragon, GR 179754, Nov. 21, 2012
REPUBLIC OF THE PHILIPPINES (through the OFFICE OF
Facts: Respondent, Jack Mondragon, sold a portion of the land in
THE SOLICITOR GENERAL), and the REGISTER OF DEEDS
dispute to his co-respondent. Petitioner filed a case to annul the
OF METRO MANILA DISTRICT III, CALOOCAN
sale arguing that respondent had no right to sell a portion of the
CITY, Respondents.
land and the sale to Regis Schmitz created a cloud upon their
title, and for quieting of title. RTC dismissed the case ruling that
FACTS: This case started with a Complaint for Quieting of Title,
since respondent was a co-owner of the land, he has the right to
Damages and Injunction filed by respondent Phil-Ville against
dispose his undivided share. CA affirmed RTC.
petitioner and the Register of Deeds of Metro Manila District III.
Respondent claims that it is the registered owner and actual ISSUE: WON respondent can dispose a portion of the said land.
possessor of sixteen (16) parcels of land in Baesa, Caloocan City
which were derived from and were part of Lot 26, Maysilo Estate RULING: Yes. In issues for quieting of title, the plaintiff need to
originally covered by Original Certificate of Title (OCT) No. 994. prove only two things: (1) the plaintiff or complainant has a legal
They alleged that based on official records of the office of or an equitable title to or interest in the real property subject of the
respondent Register of Deeds and the Land Registration action; (2) that the deed, claim, encumbrance or proceeding
Authority, petitioner was issued Transfer Certificate of Title (TCT) claimed to be casting a cloud on his title must be shown to be in
No. T-177013, covering a parcel of land situated in Caloocan fact invalid or inoperative despite its prima facie appearance of
City. validity or legal efficacy.
#57 Baha’is v Pascual (GR 169272)

ISSUE: WON petitioner’s TCT imposes a cloud on respondent Facts:


Phil-Ville’s titles to the 16 parcels of land subject matter of this On December 11, 2000, Bahais filed a complaint with RTC for
case, as provided in Article 476 of the Civil Code. quieting of title and other claims against Silverio Songcuan, his
heirs and the DENR. Bahais alleged that he is the lawful and
absolute owner of two parcels of land and that he had been in
RULING: The Court ruled in favor of respondent Phil-Ville. possession for 30 years.
Moreover, the acquisition by Atty. Dimson of the interests of the



Bureau of Lands rejected the sales applications of Bahais’s petitioner is not entitled to the possession and ownership of the
predecessors-in-interest for the lots and ordered all those in lots. They no longer have a cause of action.
privity to vacate the lots and to remove their improvements.
Jurisprudence teaches us that the decisions and orders of
DENR secretary affirmed with this decision. Recourse to the
administrative agencies, such as the Bureau of Lands, rendered
office of the President had been unavailing, so DENR issued writs
pursuant to their quasi-judicial authority, upon finality, have the
of execution.
force and binding effect of a final judgment within the purview of
the doctrine of res judicata. Accordingly, the petitioner is now
barred from challenging the validity of the final and executory
Pascual moved to dismiss saying that petitioner had no legal right
Bureau of Lands’ December 4, 1985 decision.
to file the complaint since the decision of the Bureau of Lands has
become final and executory. 58
VDA. DE AVILES v. CA
RTC denied the motion to dismiss; it was not yet final since An action to quiet title or to remove cloud may not be brought for
President's ruling on the appeal was unavailable. Respondent the purpose of settling a boundary dispute.
elevated this case to the CA, questioning the propriety of the
RTC's denial. CA set aside the RTC’s ruling. FACTS: The parties inherited their lands from their parents and
have agreed to subdivide the same amongst themselves. The
Petitioners claim that they are the owners of the fish pond which
Issue: Whether CA committed a reversible error in when it set they claim is within their area. Defendant Camilo Aviles asserted
aside RTC’s ruling a color of title by constructing a bamboo fence and moving the
earthen dikes, thereby molesting and disturbing the peaceful
possession of the plaintiffs over said portion. Petitioners brought
Held: an action to quiet title but were denied thus this case.
CA committed no reversible error. ISSUE: WON Petitioners filed the right action
RULING: An action for quieting of title may not be brought for the
purpose of settling a boundary dispute.
We find it clear that the petitioner no longer had any legal or
equitable title to or interest in the lots. The petitioner’s status as 59
possessor and owner of the lots had been settled in the final and
Fernandez vs Villegas, GR No. 200191, Aug. 20, 2014
executory December 4, 1985 decision of the Bureau of Lands that
the DENR Secretary and the OP affirmed on appeal. Thus, the FACTS: Petitioner Fernandez and Cecilia, her sister, averred that
they are the registered owners of the subject property on which


both petitioner and respondents previously lived under one roof. Respondents filed the Complaint for Annulment. In its Answer,
When their house was destroyed by a typhoon, petitioner EDC alleged that it is a buyer in good faith and for value because
transferred to a nipa hut on the same lot, while Villegas, Cecilia’s of the honest belief that the heirs of Juan are the only heirs of the
daughter-in-law, and her family were advised to relocate but, in late Apolonio.
the meantime, allowed to use a portion thereof. Respondents
erected a house thereon despite plaintiffs’ objections and, despite Issue: Whether the sale by a co-owner of a physical portion of
demands, refused to vacate and surrender possession of the an undivided property be valid.
subject property.
Held:
Yes. As a general rule, a co-owner cannot rightfully dispose of a
ISSUE: WON Fernandez’ alone can institute the ejectment suit? particular portion of a co-owned property prior to partition among
all the co-owners. However, this should not signify that the
vendee does not acquire anything at all in case a physically
segregated area of the co-owned lot is in fact sold to him. Since
RULING: YES. Article 487 of the Civil Code explicitly provides
the co-owner/vendor’s undivided interest could properly be the
that any of the co-owners may bring an action for ejectment,
object of the contract of sale, what the vendee obtains are the
because the suit is deemed to be instituted for the benefit of all. same rights as the vendor had as co-owner. Thus, the vendee
Both Lourdes and Cecilia are co-plaintiffs in the ejectment suit. steps into the shoes of the vendor as co-owner and acquires a
Thus, they share a commonality of interest and cause of action as proportionate abstract share in the property held in common.
against respondents. Hence, the lone signature of Lourdes on the
verification attached to the CA petition constituted substantial
compliance with the rules. 61
60 EXTRAORDINARY DEVELOPMENT CORPORATION (EDC), TORRES vs. LAPINID G.R. No. 187987
vs. HERMINIA F. SAMSON-BICO AND ELY B. FLESTADOS November 26, 2014
G.R. No. 191090, 13 October 2014 Facts: Petitioners and Jesus are co-owners of the disputed lot. An
Facts: action for partition of the parcels of land against the petitioners
and other co-owners were filed. A judgment was rendered based
Apolonio owned a parcel of land. When Apolonio and Maria died, on a compromise agreement where they agreed that Jesus,
the property was inherited by Juan and Irenea. When the latter Mariano and Vicente were jointly authorized to sell the said
died, the heirs of Juan and Irenea became co- owners of the properties and receive the proceeds thereof and distribute them
property. The heirs of Juan, without the consent of respondents,
to all the co-owners. The agreement was later amended to
and the heirs of Irenea executed in favor of EDC a Deed of
exclude Jesus as an authorized seller. The petitioners inspected
Absolute Sale covering the subject property. The deed was
registered and transfers the tax declaration in its name. the property and discovered that Lapinid was occupying a specific
portion of the lot by virtue of a deed of sale executed by Jesus.


No. In the instant case, Laurencia was within her hereditary
rights in selling her pro indiviso share in Lot No. 2798. However,
Issue: WON Jesus, as a co-owner, can validly sell a portion of the because the property had not yet been partitioned in accordance
property he co-owns in favor of another person. with the Rules of Court, no particular portion of the property could
be identified as yet and delineated as the object of the sale. Thus,
under Article 493 of the Civil Code, an alienation of a co-owned
Held: A co-owner has an absolute ownership of his undivided and property shall be limited to the portion which may be allotted to
pro indiviso share in the co-owned property. He has the right to (the seller) in the division upon the termination of the co-
alienate, assign and mortgage it, even to the extent of substituting ownership.
a third person in its enjoyment provided that no personal rights 63. G.R. NO. 179205 JULY 30, 2014 HEIRS OR REYNALDO
will be affected. This is evident from Art. 493 of the Civil Code. DELA ROSA, NAMELY: TEOFISTA DELA ROSA, JOSEPHINE
62 ALEJANDRINO v CA SANTIAGO AND JOSEPH DELA ROSA, PETITIONERS, VS.
MARIO A. BA TONGBACAL, IRENEO BATONGBACAL,
G.R. No. 114151. September 17, 1998 JOCELYN BA TONGBACAL, NESTOR BATONGBACAL AND
Facts: LOURDES BA TONGBACAL, RESPONDENTS.

Late spouses Alejandrino left their six children a 219-square- The Facts: The subject property consists of a 3, 750 square
meter Lot. The property should have been divided among their meter and registered under a TCT under the names of Reynaldo
children with each child having a share of 36.50 square meters. Dela Rosa and his siblings. Reynaldo offered to sell the subject
However, the estate of the Alejandrino spouses was not settled in property to Guillermo and Mario Batongbacal for P187,500.00.
accordance with the procedure outlined in the Rules of Court. Pursuant to the agreement, Reynaldo received an advance
Petitioner Mauricia (one of the children) acquired a total of 97.43 payment of P31,500.00 leaving a balance of P156,000.00. The
square meters after allegedly purchasing some of her siblings’ parties agreed that the amount of P20,000.00 as part of the
share.It turned out, however, that a third party, the private advance payment shall be paid upon the delivery of the SPA,
respondent, also purchased portions of the siblings’ property, which would authorize Reynaldo to alienate the subject property
totaling to 121.67 square meters from Mauricia’s sister, on behalf of his co-owners and siblings.
Laurencia.
Issue: ISSUE: WON the contract entered into by parties was valid.
Whether as an heir of the Alejandrino property, Laurencia may
validly sell specific portions thereof to a third party
Ruling: As a co-owner of the subject property, Reynaldo's right to
Ruling: sell, assign or mortgage his ideal share in the property held in
common is sanctioned by law. The applicable law is Article 493 of
the New Civil Code.


64)
RAUL ARAMBULO & TERESITA DELA CRUZ VS GENERO
NOLASCO & JEREMY NOLASCO
GR No. 189420, March 26, 2014
FACTS:
Petitioners filed a petition for relief alleging that all co-
owners of a 233 square meter land in Tondo, except Nolasco,
have authorized to sell their respective shares to the properties,
saying that if one or more co-owners shall withhold their consent
to the alterations in the thing owned in common, the courts may
afford adequate relief.
Nolasco responded that they did not know about the
intention to sell, because they were not called to participate in the
negotiations regarding the sale of the property.

ISSUE: Whether the respondents are withholding their consent


and whether this withholding is prejudicial to the petitioners.

HELD:
Since a co-owner is entitled to sell his undivided share, a
sale of the entire property by one co-owner without the consent of
the other co-owners is not null and void. However, only the rights
of the co-owner-seller are transferred, thereby making the buyer a
co-owner of the property.

65)
DE GUIA V. CA
FACTS:



Abejo instituted an action for recovery of possession with ISSUE: WON the appropriate remedy in the instant case is the
damages against De Guia. Abejo alleged that he is the owner of recovery of possession of the thing owned in common from the
half of the undivided portion of the property used as a fishpond third person
and that De Guia continues to possess and use such without any
contract and without paying rent to Abejo’s damage and HELD:
prejudice. De Guia also claims ownership over an undivided
No it is not. The appropriate recourse of co-owners in
portion of the fishpond and claims that partition and not recovery
of possession was the proper remedy under the circumstances. cases where their consent were not secured in a sale of the entire
property and in a mere sale of undivided shares of some of the
ISSUE: Whether a co-owner can file an ejectment case against a co-owners is an action for partition under Rule 69 of the Revised
co-owner? Rules of Court.
RULING:
Yes. Any co-owner may file an action under Article 487 67)
against another co-owner who takes exclusive possession and
asserts exclusive ownership of the entire property. However, the GAPACAN V. OMIPET
only purpose of the action is to obtain recognition of the co- 387 SCRA 383
ownership. FACTS:

Antonio and Maria Gapacan are siblings who inherited


66) unregistered land from their father, Paicat. Maria Gapacan-
Omipet eventually took over the possession of the land. Subject
BAILON-CASILAO vs CA land was then transferred to Antonio via an Affidavit and his
GR No. L-78178 April 14, 1988 family has been occupying it ever since. In 1992, Omipet filed an
action to quiet title in RTC and that she be declared the lawful
FACTS: owner. RTC ruled in Gapacan’s favor. On appeal, the CA
declared that the land is common property of both Omipet and
Rosalia and Gaudencio Bailon sold a portion of a parcel of Gapacans and ordered its partition but both parties appealed.
land to Delgado. They sold the remainder to Lanuza. Lanuza
acquired from Delgado his part of the land. Lanuza’s husband ISSUE: W/N there is a state of co-ownership
sold the whole of the land to Afable but the land was
unregistered. Heirs of Bailon filed a case of recovery of property HELD:
with damages against Afable asserting that the subject land were Yes. A state of co-ownership exists only because there is
co-owned by them. unity of the object or property and plurality of subjects.



68) could repurchase the property. During the redemption period,
QUINTOS V. NICOLAS Rustico Adille repurchased the property by himself alone at
GR No. 210252, June 16, 2014 his own expense, and after that, he executed a deed of extra-
FACTS: judicial partition representing himself to be the only heir and
child of his mother Felisa. Consequently, he was able to secure
Petitioners filed a complaint for Quieting of Title against title in his name alone. His half-siblings, herein respondents, filed
respondents wherein they averred that they have been in a case for partition and accounting claiming that Rustico
adverse, open, continuous, and uninterrupted possession of the was only a trustee on an implied trust when he redeemed the
property for over four (4) decades and are, thus, entitled to property, and thus, he cannot claim exclusive ownership of
equitable title thereto. Respondents countered that petitioners’ the entire property.
cause of action was already barred by estoppel.
RTC dismissed the case holding that it did not find merit in ISSUES:
petitioners’ claim that they have acquired title over the property
through acquisitive prescription. CA affirmed RTC ruling. 1) Whether or not a co-owner may acquire exclusive
ownership over the property held in common.
ISSUE: 2) Whether or nor Rustico had constituted himself a
W/N the Petitioners’ Complaint for Quieting of Title in their negotiorum gestor
land was duly complied with
HELD:
HELD:
No it was not. Petitioners were unable to comply with first 1) No. The right to repurchase may be exercised by a co-
indispensable requisite to make an action for Quieting of Title owner with respect to his share alone. Although Rustico
prosper which is proving that (1) they had legal or equitable title Adille redeemed the property in its entirety, shouldering the
to or interest in the real property. expenses did not make him the owner of all of it.

2) Yes. The petitioner, in taking over the property, did so on


69 behalf of his co-heirs, in which event, he had constituted
himself a negotiorum gestor under Art 2144 of the Civil
ADILLE vs. COURT OF APPEALS, 157 SCRA 455, January 29, Code, or for his exclusive benefit, in which case, he is
1988 guilty of fraud, and must act as trustee, the respondents
being the beneficiaries, pursuant to Art 1456.
FACTS: The property in dispute was originally owned by Felisa
Alzul who got married twice. Her child in the first marriage
was petitioner Rustico Adile and her children in the second
marriage were respondents Emetria Asejo et al.
During her lifetime, Felisa Alzul sold the property in pacto de retro
with a three-year repurchase period. Felisa died before she

70
Sanchez v CA

Facts: Lilia Sanchez constructed a house on a 76-square meter


lot. The lot was registered with six co-owners, including petitioner.
Later, the lot was registered to Teria, respondent, by virtue of a
Deed of Absolute Sale supposed to have been executed by all six
(6) co-owners in her favor. Petitioner claimed that she did not affix
her signature on the document and subsequently refused to
vacate the lot. MeTC declared the sale was valid only to the
extent of 5/6 of the lot and the other 1/6 remaining as the property
of petitioner, on account of her signature in the Deed of Absolute
Sale having been established as a forgery. A writ of execution
was issued and a Notice to Vacate was served by the sheriff
upon petitioner who however refused to heed the Notice.

Issue: WON the sale is valid as to petitioner’s share?

Ruling: No. Although assigned an aliquot but abstract part of the


property, the metes and bounds of petitioners lot has not been
designated. As she was not a party to the Deed of Absolute Sale
voluntarily entered into by the other co-owners, her right to 1/6 of
the property must be respected. Partition needs to be effected to
protect her right to her definite share and determine the
boundaries of her property. Such partition must be done without
prejudice to the rights of private respondent Virginia Teria as
buyer of the 5/6 portion of the lot under dispute.



71 other co-owners and that they have been categorically advised of
Pangan v CA the exclusive claim he is making to the property in question.
There was no adequate notice by the petitioners to the private
G.R. No. L-39299
respondent of the rejection of her claim to her share in the subject
October 18, 1988 property.

FACT: 72. Heirs of Restar –vs-Heirs of Cichon ~Diana

The petitioner filed an application for the registration of the land in 72)
Heirs of Restar –vs-Heirs of Cichon
their names by virtue of their continuous and exclusive
possession by them which application was approved. The private FACTS:
Flores took over the land that he and his siblings inherited
respondent then filed a petition to set aside the said decision on from their father but he died intestate. Flores’ siblings/co-owners
the ground that she as an heir too, is entitled to the share of the filed a Complaint for partition of the land against Flores’ heirs.
The latter claimed that they had been in possession of the lot as
property. However the petition was dissmissed on the ground that owners for more than thirty (30) years. The RTC of Aklan ruled in
the private respondent right over the property had been forfeited favor of Flores’ heirs holding that there was repudiation of co-
ownership and that the lot was acquired by prescription. The CA
by extinctive prescription. On appeal, the decision was reversed. reversed the RTC decision on appeal.

ISSUE:
W/N the Heirs’ of Flores acquired ownership through
ISSUE: Whether the private respondent is entitled to the share of prescription despite co-ownership
the property.
HELD:
Yes, there has been acquisition through prescription.
While the action to demand partition of a co-owned
property does not prescribe, a co-owner may acquire co-
HELD: ownership thereof by prescription where there exists a clear
repudiation of the co-ownership, and the co-owners are apprised
Yes. Private respondent as an heir is a co-owner of the property. of the claim of adverse and exclusive ownership.
For title to prescribe in favor of the co-owner, however, there
must be a clear showing that he has repudiated the claims of the



TORAYNO – 73 Issue: Whether or not petitioners’ action for partition is already
EPITACIO DELIMA, PACIANO DELIMA, FIDEL DELIMA, ET AL. barred by the statutory period provided by law?
VS. COURT OF APPEALS
G.R. NO. L-46296 201 SCRA 641 SEPTEMBER 24, 1991 Held: Yes. Being that Galileo was holding the property in
PONENTE: MEDIALDEA, J. representation of the co-owners; he was therefore acting as an
administrator who took care of the property yet still having the
Doctrine: No prescription shall run in favor of a co-owner against ultimate obligation to deliver the property to his co-owners.
his co-owners or co-heirs as long as he expressly or impliedly However this rule shall no longer apply when one of the co-
recognizes the co-ownership. The exception however is that the owners begin to claim the absolute and exclusive ownership and
from the moment one of the co-owners claims that he is the denies the others any share therein. Since Galileo, having
absolute and exclusive owner of the properties and denies the executed a deed of partition and obtained subsequent to that the
others any share therein, the question is no longer one of partition cancellation of the old title and the creation of a new one wherein
but of ownership. he appears as the new owner of the property, he thereby in effect
denied and repudiated the ownership of the other co-owners over
Facts: Lino Delima acquired a lot by sale on installments from the their shares. From this act, the statute of limitations started to run.
government. After his demise in 1921 he had his three brothers
and a sister listed as his heirs, Eulalio, Juanita, Galileo, and 74
Vicente. A new TCT was issued. Galileo executed an affidavit of Mariategui v. C.A.
Extra-judicial Declaration of Heirs adjudicating to himself the 205 SCRA 337

subject property excluding the other heirs. He declared the lot to FACTS:
be of his own and paid for its taxes. Eulalio and Juanita filed an
Mariategui died without a will. During his lifetime, he contracted
action for reconveyance and partition of property and for the three (3) marriages. Lupo's descendants by his first and second
annulment of the certificate of title issued. marriages executed a deed of extrajudicial partition whereby they
adjudicated unto themselves lots in the Muntinglupa Estate.
Thereafter, an Original Certiicate of Title (OCT) was issued in
their name. Subsequently, separate transfer certificates of title


were issued after subdivision. Lupo's children by his third ISSUE:
marriage filed an amended complaint. They pray for partition of W/N a partition of the subject property appropriate in the
the estate of their deceased father and annulment of the deed of instant case
extrajudicial partition.
HELD:
ISSUE: The petition was denied.
Whether or not prescription barred private respondents' right to The first phase of partition is taken up with the
demand the partition of the estate of Mariategui. determination of whether or not a co-ownership exists, and a
partition is proper and may be made by voluntary agreement of all
HELD: the parties interested in the property. This phase may end with a
No. Prescription of an action for partition does not lie except when declaration that plaintiff is not entitled to have a partition either
the co-ownership is properly repudiated by the co-owner. No valid because co-ownership does not exist, or that partition is legally
repudiation was made by petitioners to the prejudice of private prohibited.
respondents. Assuming petitioners' registration of the subject lot
was an act of repudiation of the co-ownership, prescription had
not yet set in when private respondents filed the present action
for partition. Petitioners' registration of the properties in their 76)
names did not operate as a valid repudiation of the co-ownership. CRUZ V. CATAPANG
GR No. 164110, February 12, 2008
FACTS:
75. Lacbayan vs. Samoy, G.R. No. 165427, March 21, 2011 Petitioners are co-owners of a land in Taal, Batangas. One
75) of the petitioners gave respondent permission to construct her
LACBAYAN VS. SAMOY (respondent’s) house adjacent to the co-owned property. The
G.R. No. 165427, March 21, 2011 house encroached on the subject property. Petitioners filed an
FACTS: action for forcible entry against respondent in the MTC which
Respondent, although married, had an affair with ruled in their favor. RTC affirmed MTC’s ruling on appeal. The CA
petitioner. Five parcels of land were acquired and registered reversed RTC ruling in favor of respondent.
under the petitioner’s and respondent’s name as husband and
wife for a business venture. Eventually they decided to part ways ISSUE: p
and agreed to divide said properties and terminate their business WON the consent given by a co-owner of a parcel of land
partnership. The respondent and petitioner disagreed on the to a person to construct a house on the co-owned property
terms of the partition of subject parcels. warrants the dismissal of a forcible entry case
Petitioner filed a complaint for judicial partitioning of the
said properties. The RTC dismissed the complaint for lack of HELD:
merit and was then elevated to the CA. No. A co-owner cannot give valid consent to another to
build a house on the co-owned property, which is an act
tantamount to devoting the property to his/her exclusive use, to


the prejudice of the co-ownership. The construction of a house on G.R. No. 179914 June 16, 2014
the co-owned property is an act of dominion. Therefore, it is an
alteration falling under Article 491 of the Civil Code. There being SPOUSES REYNALDO AND HILLY G.
no consent from all co-owners, respondent had no right to SOMBILON, Petitioners,
construct her house on the co-owned property. vs.
ATTY. REY FERDINAND GARAY AND PHILIPPINE NATIONAL
BANK, Respondents.
#77 SANTOS v. LUSTRE G.R. No. 151016
FACTS

FACTS: Spouses Sombilon were the previous owners of a property, which


was foreclosed and sold at public auction. PNB was the winning
In Civil Case No. 1330, both heirs of Dominga Lustre, filed with bidder. The spouses failed to redeem the property. In 2005, they
the RTC, a complaint for Declaration of the Inexistence of sought the help of Atty. Garay. Both partieswere informed by PNB
Contract, Annulment of Title, Conveyance and Damages against that the property could be purchased at P2,938,000.00. The
Froilan Santos, son of the appellant spouses. In another Civil following day, Atty. Garay bought the property for himself by
case, filed by Lustre’s other heirs against the parties of this case, making a down payment of P587,600.00. Upon learning, spouses
they averred that the sale of the property to Natividad Santos was Sombilon offered to buy back the property from
simulated, spurious or fake, and that they discovered that the PNB. Subsequently, when a Final Deed of Conveyance was
spouses Santos transferred the property to Froilan Santos when issued in favor of PNB, the latter decided to approve the
an ejectment suit was filed against them. purchase offer of Atty. Garay since spouses Sombilon failed to
make the required down payment. PNB filed an Ex-Parte Petition
ISSUE: for Issuance of a Writ of Possession, which Judge Venadas, Sr.
held in abeyance.
Whether the action for reconveyance on the ground that the
certificate of title was obtained by means of a fictitious deed of ISSUES
sale is virtually an action for the declaration of its nullity
(1) Whether or not Judge Venadas, Sr. committed grave abuse of
discretion in holding in abeyance the implementation of the Writ
RULING: of Possession;
Yes. When an action for reconveyance is filed, it would be in the HELD
nature of a suit for quieting of title, an action that is
imprescriptible. It follows then that the respondents’ present Yes, he committed grave abuse of discretion because PNB, as
action is not barred by laches. the registered owner, is entitled to the possession of the subject
property as a matter of right.The redemption period had long
78
lapsed when PNB applied for the issuance of the Writ of


Possession.In fact, the title over the subject property had already W/N respondent are entitled to issuance of writ or possession
been consolidated in PNB’s name. The claim that the sale
between PNB and Atty. Garay was invalid as it was done in
violation of paragraph 5, Article 1491 of the Civil Code iswrong. Ruling:
First, it was not a ground to defer the issuance of the Writ of
Possession. Second, it does not affect PNB’s right to possess the Yes. If the purchaser is the mortgagee or third party during
subject property. redemption period, writ may issue ex-parte. The remedy is
available to subsequent purchaser but only after hearing and after
determining that subject property is still in the possession of the
mortgagor. If the property is in the possession of the mortgagor, a
VISTAL – 79 writ of possession could be issued. There is no dispute that
Javate v. Sps. Tiotuico petitioner is in possession of the property thus, issuance of the
writ is proper. To require subsequent purchaser to file a separate
G.R. No. 187606 case of ejectment will only prolong and unduly deny possession
March 9, 2015 of property which already bought.

Peralta, J..
80)

Facts of the case; DELA CRUZ V. SPS HERMANO


GR No. 160914, March 25, 2015
FACTS:
Petitioner’s land was foreclosed by the bank for failure to pay his Respondent spouses, the registered owner of subject
obligation. He failed to redeem it within the given period, thus house and lot, filed an ejectment suit against petitioner.
Respondent claims that said property is used as a rest/vacation
ownership was consolidated with the bank. Respondent then
house. In his defense, petitioner alleged that the property was
bought the subject property from the bank and petition the court
already sold by respondent to a certain Benitez who sold the
for a writ of possession, which was granted. Petitioner contend same to herein petitioner. Also, the case at bar is not one of an
that only the bank has the right to issuance of writ of possession, unlawful detainer or forcible entry to which an ejectment suit is
respondents as a subsequent buyer must resort to appropriate based.
judicial remedy, which is ejectment or accionreinvindicatoria in
order to gain possession. ISSUE:
Whether the ejectment suit will prosper?

Issue: HELD:



No, it will not. The Supreme Court agrees that this was an
action for ejectment in the nature of accion reivindicatoria, the ISSUE:
case was actually for forcible entry and sufficient in form. Whether or not being a builder in good faith matters under article
However, the respondents failed to provide sufficient evidence in 448.
their allegation of prior physical possession thereof, which is an
element in forcible entry. HELD:
– 81
NO. Article 448, of the Civil Code refers to a piece of land
whose ownership is claimed by two or more parties, one of whom
PNB V. DE JESUS has built some works (or sown or planted something) and not to a
411 SCRA 557 case where the owner of the land is the builder, sower, or planter
who then later loses ownership of the land by sale or otherwise
FACTS: for, elsewise stated, “where the true owner himself is the builder
On 10 June 1995, respondent filed a complaint against of works on his own land, the issue of good faith or bad faith is
petitioner before the Regional Trial Court of Occidental Mindoro entirely irrelevant.”
for recovery of ownership and possession, with damages, over
the questioned property. In his complaint, respondent stated that
82
he had acquired a parcel of land situated in Mamburao,
Occidental Mindoro, with an area of 1,144 square meters covered PARILLA v. PILAR
by TCT No. T-17197, and that on 26 March 1993, he had caused
G.R. No. 167680, November 30, 2006
a verification survey of the property and discovered that the
northern portion of the lot was being encroached upon by a FACTS:
building of petitioner to the extent of 124 square meters. Despite
Petitioners, as dealers of Pilipinas Shell Petroleum Corporation
two letters of demand sent by respondent, petitioner failed and
(Pilipinas Shell), have been in possession of a parcel of land
refused to vacate the area.
which was leased to them by the respondent. When the lease
Petitioner, in its answer, asserted that when it acquired the
contract expired, and despite demands to vacate, petitioners
lot and the building sometime in 1981 from then Mayor
remained in possession of the property on which they built
Bienvenido Ignacio, the encroachment already was in existence
and to remedy the situation, Mayor Ignacio offered to sell the area improvements. Hence, respondent filed a complaint for ejectment
in question (which then also belonged to Ignacio) to petitioner at before the MTC which ordered the petitioners to vacate and to
P100.00 per square meter which offer the latter claimed to have pay respondent a reasonable compensation for the use of the
accepted. The sale, however, did not materialize when, without property. It also ordered respondent to reimburse the petitioners
the knowledge and consent of petitioner, Mayor Ignacio later the amount Two Million Pesos representing the value of the
mortgaged the lot to the Development Bank of the Philippines. improvements introduced on the property.
He also contends that he is a builder in good faith.


RTC affirmed the MTC‘s Decision. However, the CA set aside the – 83
lower court’s decision.
ABALOS VS. HEIRS OF TORIO
ISSUES:
FACTS:
Whether or not the petitioners are entitled to reimbursement for
the improvements being builders in good faith under Art. 453 of On July 24, 1996, herein respondents filed a Complaint for
the Civil Code. Recovery of Possession and Damages with the Municipal Trial
Court (MTC) of Binmaley, Pangasinan against Jaime Abalos
HELD:
(Jaime) and the spouses Felix and Consuelo Salazar.
Article 448 covers only cases in which the builders, sowers or Respondents contended that they are the children and heirs of
planters believe themselves to be owners of the land or, at least, one Vicente Torio (Vicente) who died intestate, leaving behind a
have a claim of title thereto, but not when the interest is merely parcel of land measuring 2,950 square meters, more or less,
which is located at San Isidro Norte, Pangasinan. During the
that of a holder, such as a mere tenant, agent or usufructuary. A
lifetime of Vicente and through his tolerance petitioners. were
tenant cannot be said to be a builder in good faith as he has no
allowed to stay and build their respective houses on the subject
pretension to be owner. parcel of land. Even after the death of Vicente, herein
It is Article 1678 of the New Civil Code which applies to the respondents allowed The petitioners to remain on the disputed
present case. However, the petitioners’ claim for reimbursement lot. However, in 1985, respondents asked Jaime and the Spouses
of the alleged entire value of the improvements still does not thus Salazar to vacate the subject lot, but they refused to heed the
demand of respondents forcing respondents to file the complaint.
lie under Article 1678 there being no substantial evidence, e.g.,
receipts or other documentary evidence detailing costs of Jaime and the Spouses Salazar filed their Answer with
construction. Besides, of the structures they originally built, only Counterclaim, denying the material allegations in the Complaint
the bodega-like, sari-sari store, and the parking lot now exist. and asserting in their Special and Affirmative Defenses that
At all events, under Article 1678, it is the lessor who is given the respondents' cause of action is barred by acquisitive prescription.
Petitioners claim the court has no jurisdiction over the nature of
option, upon termination of the lease contract, either to
the action and the persons of the defendant. They also alleged
appropriate the useful improvements by paying one-half of their that they are in actual, continuous and peaceful possession of the
value at that time, or to allow the lessee to remove the subject lot as owners since time immemorial. They also said that
improvements. This option solely belongs to the lessor as the law they have been paying real property taxes and have been
is explicit that ―[s]hould the lessor refuse to reimburse said introducing improvements on the said land.
amount, the lessee may remove the improvements, even though
the principal thing may suffer damage thereby. It appears that the ISSUES:
lessor has opted not to reimburse. (1) Whether or not the petitioners and their predecessors-in-
interest possessed the disputed lot in the concept of an owner.



(2) Whether or not the possession of the petitioners is by mere In the case at bar, the CA correctly held that even if the character
tolerance of respondents and their predecessors-in-interest. of petitioners' possession of the subject property had become
(3) Whether or not the petitioners qualify to own the land through adverse, as evidenced by their declaration of the same for tax
acquisitive prescription. purposes under the names of their predecessors-in-interest, their
possession still falls short of the required period of thirty (30)
HELD: years in cases of extraordinary acquisitive prescription. Records
show that the earliest Tax Declaration in the name of petitioners
(1) No. was in 1974. Reckoned from such date, the thirty-year period was
completed in 2004. However, herein respondents' complaint was
In the instant case, it is clear that during their possession of the filed in 1996, effectively interrupting petitioners' possession upon
property in question, petitioners acknowledged ownership thereof service of summons on them.24 Thus, petitioners possession
by the immediate predecessor-in-interest of respondents. This is also did not ripen into ownership, because they failed to meet the
clearly shown by the Tax Declaration in the name of Jaime for the required statutory period of extraordinary prescription.
year 1984 wherein it contains a statement admitting that Jaime's
house was built on the land of Vicente, respondents' immediate
predecessor-in-interest.21 Petitioners never disputed such an
acknowledgment. Thus, having knowledge that they nor their 84
predecessors-in-interest are not the owners of the disputed lot, MANGASER V. UGAY
petitioners' possession could not be deemed as possession in GR No. 204926, December 3, 2014
good faith as to enable them to acquire the subject land by
ordinary prescription. FACTS:
Petitioner Anacleto Mangaser filed an Action of Forcible
(2) Yes. Entry against Respondent Ugay. However, the latter contended
In this respect, the Court agrees with the CA that petitioners' that Petitioner was unable to prove prior physical possession over
possession of the lot in question was by mere tolerance of subject property.
respondents and their predecessors-in-interest. Acts of
possessory character executed due to license or by mere ISSUE:
tolerance of the owner are inadequate for purposes of acquisitive W/N the Action for Forcible Entry would prosper
prescription.Possession, to constitute the foundation of a
prescriptive right, must be en concepto de dueo, or, to use the HELD:
common law equivalent of the term, that possession should be Yes. Physical possession is not an indispensible
adverse, if not, such possessory acts, no matter how long, do not requirement. Possession can also be acquired by juridical acts.
start the running of the period of prescription. These are acts to which the law gives the force of acts of
possession. Examples of these are donations, succession,
(3) No. execution and registration of public instruments, inscription of



possessory information titles and the like. It is sufficient that – 85
petitioner was able to subject the property to the action of his will.
Suarez vs. Emboy, G.R. No. 187944, March 12, 2014

Facts:

A parcel of land was partitioned into 5 among the heirs of the


Carlos and Asuncion. Lot No. 1907-A-2 was occupied by Felix
and Marilou Emboy, who were claiming that they inherited it from
their mother Claudia Emboy, who inherited it from her parents.
Felix and Marilou were asked by their cousins to vacate the lot
and transfer to Lot No. 1907-A-5 but they refused to comply and
insisted that Claudia's inheritance pertained to Lot No. 1907-A-2.
In 2004, Felix and Marilou received a demand letter from
Carmencita requiring them to vacate the lot andinformed them
that they have purchased the same. Felix and Marilou argued that
the complaint for unlawful detainer was fundamentally inadequate
since there was no specific allegation as to when and how
possession by tolerance of them began.

Issue: Whether or not the complaint for unlawful detainer was


inadequate.

Held:

In a complaint for unlawful detainer, there are requisites that are


needed to be alleged. One of such requirements include, “initially,
possession of property by the defendant was by contract with or
by tolerance of the plaintiff”. In this case, the first requisite was
absent. Carmencita failed to clearly allege and prove how Emboy
entered the lot and constructed a house upon it. Hence, the
complaint should not have been for unlawful detainer and the CA
did not commit an error in dismissing Carmencita's complaint



The spouses Dela Cruz are able to establish by preponderance of
evidence that they are the rightful possessors of the property. The
spouses Dela Cruz were able to prove by preponderance of
86 evidence that they are the owners of the lot. Their allegation is
supported by a copy of the decision in Land Registration Case
G.R. No. 176055 March 17, 2014 No. 9511, title of land issued to Teodora and extra-judicial
settlement of the estate of Teodora conveying the said property to
SPOUSES EDMUNDO DELA CRUZ and AMELIA CONCIO- Amelia. Petitioners are better entitled to the material possession
DELA CRUZ, Petitioners, of the subject property. As its present owners, they have a right to
vs. the possession of the property which is one of the attributes of
SPOUSES RUFINO R. CAPCO AND MARTY1 C. ownership.
CAPCO, Respondents.

FACTS:
CASE No. 87
Spouses Dela Cruz filed a Complaint for Unlawful Detainer PIEDAD vs. GURIEZA, G.R. No. 207525, June 18, 2014
against the spouses Capco before the MeTC of Pateros. They
alleged that Teodora T. Concio, mother of petitioner Amelia Facts:
Concio-Dela Cruz, acquired ownership over a piece of land by Piedad alleged that he is the absolute owner of the 1/3 portion of
virtue of a decision of RTC of Pasig. Petitioners said that Teodora land which he acquired from his late father, through a Deed of
tolerated the spouses Capco’s occupation thereof and conveyed Confirmation. Before migrating to Hawaii, he built a bungalow on
the lot and assigned Spouses Gurieza as the last caretakers.
it to them due to their close ties as neighbors and relatives.
However, Sps. Gurieza allegedly had the lot declared under their
Intending to construct a house thereon and use it for business,
name for tax purposes. He sent his daughter to the country to
peititioners asked them to vacate the land but the respondents demand the spouses to vacate and initiate a complaint. Sps.
refused. In their answer, respondents pointed out that the Gurieza maintained they acquired the lot through acquisitive
complaint is defective for failing to allege the exact metes and prescription. CA ruled that Gurieza, whom Bonifacio recognized
bounds of the property and that there was no title thereto to show as one of the heirs of the subject lot, continues to be a co-owner
such ownership. thereof, because the lot was never partitioned since Gurieza
never signed the Deed of Confirmation.
ISSUE:
Whether or not the spouses Dela Cruz have the better right to Issue:
possess the property. Whether or not the ejectment suit would prosper?

HELD: Held: Yes. An ejectment case, based on the allegation of


possession by tolerance, falls under the category of unlawful
detainer. Where the plaintiff allows the defendant to use his


property by tolerance without any contract, the defendant is On appeal, the Regional Trial Court (RTC) reversed the
necessarily bound by an implied promise that he will vacate on judgment of the MTCC, and dismissed the complaint holding that
demand, failing which, an action for unlawful detainer will lie. the summary proceeding for ejectment was not proper. Serious
question of ownership of the disputed property was involved and
which cannot be determined in a summary proceeding for
ejectment.

The CA affirmed the decision of the RTC on the ground


that the respondent was either a co-owner or an assignee holding
the right of possession over the disputed property; that the
respondent became a co-owner along with Eliseo and his co-
heirs, giving him the right to participate in the partition of the
estate owned in common by them; that because the respondent
88, FEU. QUIJANO vs ATTY. DARYLL A. AMANTE was not given any notice of the project of partition or of the
intention to effect the partition, the partition made by the petitioner
FACTS: and her co-heirs did not bind him.

The petitioner and her siblings, namely: Eliseo, Jose and ISSUE:
Gloria, inherited from their father a parcel of land. Prior to any
partition among the heirs, Eliseo sold a portion of his share to Whether or not review on certiorari is correctly sought by
respondent. the petitioner.

Fe, Eliseo, Jose and Gloria executed a deed of RULING:


extrajudicial partition to divide their father's estate among
themselves. The partition resulted in the portions earlier sold by The petition for review on certiorari lacks merit. To show
Eliseo to the respondent being adjudicated to the petitioner that the possession was initially lawful, the basis of such lawful
instead of to Eliseo. The respondent's occupation had become possession must then be established. The acts of tolerance must
illegal following his refusal to vacate despite repeated demands. be proved, for bare allegation of tolerance did not suffice. At least,
Hence, this action for unlawful detainer. the petitioner should show the overt acts indicative of her co-
heirs' permission for him to occupy the disputed property. But she
The MTCC rendered its decision in favor of the petitioner. did not adduce such evidence. Instead, she appeared to be
The parcel of land left by their father had not yet been partitioned, herself not clear and definite as to the respondent’s possession of
rendering Eliseo a mere co-owner of the undivided estate who the disputed property being merely tolerated by Eliseo.
had no right to dispose of a definite portion thereof.
In contrast, the respondent consistently stood firm on his
assertion that his possession of the disputed property was in the
concept of an owner, not by the mere tolerance of Eliseo and


actually presented the deeds of sale transferring ownership of the causes of action. With the identity of the parties, subject matter,
property to him. Unlawful detainer action is dismissed for being and cause of action between the first and second ejectment suit,
an improper remedy. it cannot thus be seriously doubted that the final and executory
judgment in the first case had already barred the resolution of the
Case Number 89: second. Hence, the second ejectment complaint of dela Llana is
dimissed.
De Leon v. Dela Llana
GR No. 212277
February 11, 2015 90) Bunyi v. Factor

FACTS:
The case stemmed from an unlawful detainer complaint filed by FACTS:
Gilbert dela Llana against Robert de Leon. The said complaint Respondent Fe S. Factor and Gloria Factor-Labao are co-owners
averred that sometime in 1999, dela Llana through an undated of an 18-hectare piece of land. Gloria Factor-Labao who was
contract of lease, leased a portion of a 541 square-meter married to Ruben Labao died thus the administration and
property, registered in his name, to De Leon which the latter management of the Factor compound including the subject rest
intended use as a lottery. Upon failure to pay rental, Dela Llana house, passed on to respondent Fe S. Factor as co-owner of the
filed an ejectment suit to De Leon, but the latter posited that lease property. Factor, considering that Labao was sickly and had no
contract was fraudulent and not binding. The lower court ruled in means of income, respondent allowed him to stay at the rest
favor of Del Leon where such decision was final and executory. house for brief, transient and intermittent visits as a guest of the
De Llana filed a second complaint for unlawful detainer. De Leon Factor family. Labao was married to Bunyi. Eventually, he died.
raised the defense of res judicata, particularly averring that the Petitioners stole the personal properties of said houses and
second ejectment complaint should be dismissed given that it occupied the premises. Hence, respondent Fe S. Factor filed a
was already barred by prior judgment. complaint for forcible entry against herein petitioners Precy Bunyi
and Mila Bunyi.
ISSUE:
Whether the principle of res judicata applies – that is, whether the ISSUE:
second ejectment complaint was barred by prior judgment. WON the petioner has a better right of physical and material
possesion of the subject property.
RULING:
RULING:
The principle of res judicata applies. There is a bar by prior For one to be considered in possession, one need not have
judgment where there is identity of parties, subject matter, and actual or physical occupation of every square inch of the property
causes of action between the first case where the judgment was at all times. Possession can be acquired not only by material
rendered and the second case that is sought to be barred. There occupation, but also by the fact that a thing is subject to the
is conclusiveness of judgment, on the other hand, where there is action of one’s will or by the proper acts and legal formalities
identity of parties in the first and second cases, but no identity of


established for acquiring such right. Possession can be acquired police. He admitted that he was about to convey the package to
by juridical acts. a certain Gonzales. Appellant denied that the package in his
possession contained marijuana, but the Trial Court rejected his
contention. Taking one step further, the appellate court went on to
91 declare that being mala prohibita, one commits the crime under
PEOPLE vs. PENAFLORIDA RA No. 6425 by mere possession of a prohibited drug without
GR NO. 175604 legal authority. Intent, motive or knowledge thereof is not
April 10, 2008 necessary.

Facts:
SPO3 Vicente Competente of PNP Station Camarines Sur 92
received a tip from an asset that a bundle of marijuana was MARK SOLEDAD y CRISTOBAL,
being transported by appellant. A team of police officers was - versus -
organized for the operation. The team boarded the police mobile
car and proceeded to Sitio Nasulan in Barangay Huyon-huyon. PEOPLE OF THE PHILIPPINES,
They overtook appellant who was on a bicycle. The police officers The facts of the case, as narrated by the CA, are as follows:
flagged appellant down and found marijuana wrapped in a
cellophane and newspaper together with other grocery items. The Sometime in June 2004, private complainant Henry C. Yu
amount of P1550.00 was also found in appellant’s possession. received a call on his mobile phone from a certain Tess or Juliet
The police officers confiscated the items. The appellant alleged Villar ( Rochelle Bagaporo), a credit card agent, who offered a
that he was arbitrarily arrested by the officers. TC ruled that there Citifinancing loan assistance at a low interest rate. Yu was invited
was violation of Section 4, Article II of the Dangerous Drugs Act. by Bagaporo to his office, where he was introduced to her boss.
Appellant asserts that there was doubt as to his possession of the In compliance of the requriements for the loan, complainant
marijuana and that he was arbitrarily arrested. submitted various documents, such as his Globe handyphone
original platinum gold card, identification cards and statements of
Issue: Whether or not the contention of the appellant is tenable? accounts. Subsequently, private complainant followed up his loan
status but he failed to get in touch with either [petitioner] or
Ruling: Ronald Gobenchiong. Petitioner was thus charged with Violation
of Section 9(e), R.A. No. 8484 for possessing a counterfeit
No. The warrantless arrest is justified. There was a clear violation access device or access device fraudulently applied for.
of Article II, Section 4 of RA No. 6425, as amended by RA No
7659, which specifically covers the sale, administration, delivery,
distribution and transportation of prohibited drugs. In the instant ISSUE: Whether or not petitioner was legally in possession of the
case, appellant was riding his bicycle when he was caught by the credit card subject of the case.


No merit Whether or not the Regional Trial Court erred when it ordered the
transfer of possession seized to Chua when the latter filed an
action for replevin.
The trial court convicted petitioner of possession of the credit card
fraudulently applied for, penalized by R.A. No. 8484. The law, Held:
however, does not define the word possession. Thus, we use the
Yes. The Regional Trial Court of Cebu Branch VIII erred when it
term as defined in Article 523 of the Civil Code, that is, ordered the transfer of possession of the property seized to Chua
possession is the holding of a thing or the enjoyment of a right. when the latter filed the action for replevin. It should have
The acquisition of possession involves two elements: the corpus dismissed the case since by virtue of the "provisional dismissal",
or the material holding of the thing, and the animus possidendi or of the carnapping case there is still a probability that a criminal
the intent to possess it. case would be filed, hence a conflict in jurisdiction could still
arise. The basic principle that a judge who presides in one court
cannot annul or modify the orders issued by another branch of
93 - CHUA-BURCE vs. CA the same court because they are co-equal and independent
[G.R. No. 109595. April 27, 2000] bodies acting coordinately, must always be adhered to.

Facts:
94
GABRIEL
Judge Lauro V. Francisco of RTC Cebu City issued a search
VS
warrant directing the immediate search of the premises of R.R.
CRISOLOGO
Construction and the seizure of an Isuzu dump truck. Canoy
G.R. NO. 204626, JUNE 9, 2014
seized the vehicle and took custody, where later on a writ of
____________________________________________________
replevin was filed to recovery it. Chua alleged that he had a lawful
_________________________________
ownership and possession of the vehicle; that he had not sold the
vehicle to anyone; that he had not stolen nor carnapped it, and FACTS:
that he had never been charged of the crime of carnapping or any
other crime for that matter. Meanwhile, a case for carnapping Crisologo alleged that she was the registered owner of two
against Chua pending preliminary investigation before the City parcels of land. She discovered that petitioners unlawfully entered
Fiscal’s Office of Cebu City. The CA reversed the RTC decision and occupied her properties by stealth, force and without her prior
and ordered the dismissal of the Replevin action, and directed consent and knowledge and constructed houses on the said lots.
that possession of the subject vehicle be restored to Canoy. Upon the discovery, Atty Carmelita Crisologo, the daughter,
demanded that petitioners vacate the premises and remove their
structures thereon. Petitioners refused to vacate the said
Issue: properties despite repeated demands.

ISSUE: ISSUE:
Whether or not Bernard is the lawful possessor of the casino
Whether the petitioners have a better right of possession over the subject
chips entitling him to collect from the casino and award of
parcels of land.
damages.
HELD:
RULING:
Crisologo has a better right of possession over the subject
parcels of land. The AccionPubliciana is an ordinary civil proceeding There is no basis to suppose that the casino chips found in
to determine the better right of possession of realty independently Ludwin and Deoven’s possession were stolen; petitioner acted
of title. It refers to an ejectment suit filed after the expiration of arbitrarily in confiscating the same without basis. If it cannot be
one & year from the accrual of the cause of action or from the proved, in the first place, that Cabrera stole the chips, then there
unlawful withholding of possession of the realty. The objective of the is no more reason to suppose that Ludwin and Deoven were
plaintiffs in accion publiciana is to recover possession only, not dealing in or possessed stolen goods; unless the independent
ownership.
fact that Cabrera stole the chips can be proved, it cannot be said
that they must be confiscated when found to be in Ludwin and
95 Deoven’s possession. If Subic Bay Legend cannot prove its loss,
Subic Bay Legend Resorts and Casinos Inc. vs. Fernandez then Article 599 cannot apply; that the presumptions that the
chips were exchanged for value remains.
September 29, 2014 G.R. No. 193426=
96
FACTS:
Penta Pacific Realty Corp. vs Ley Construction and
On July 1, 1997, Bernard Fernandez, a brother of Ludwig and Development Corp
Deoven, filed a complaint for recovery of sum of money and Nov. 24, 2014, GR No. 161589
damages against the company. According to him, he went to the
casino on June 13, 1997; he handed to his brothers $6,000.00 Facts: Penta Pacific leased its properties to Ley Construction.
worth of chips belonging to him, for use at the casino. Thereat, Both parties then entered into a contract to sell. Ley Construction
the company personnel accosted his brothers and confiscated his failed to pay its amortizations prompting Penta Pacific to file an
casino chips worth $5,900.00, and failed to return the same to action for ejectment. The CA affirmed the ruling of the RTC that
him despite demand. Brothers Deoven and Ludwig Fernandez the MeTC had no jurisdiction over the case.
was accused of stealing casino chips from Subic Bay Legend Issue: Whether the complaint was for unlawful detainer, or accion
Resorts and Casinos Inc. They were made to confess that the publiciana, or accion reivindicatoria.
chips were supplied by a casino employee, Michael Cabrera.


Ruling: The SC ruled that a defendant’s claim of possession de 97
jure or his averment of ownership does not render the ejectment
Javier vs Lomuntad
suit either accion publiciana or accion reivindicatoria. The suit
remains accion interdictal, a summary proceeding that can G.R 203760
proceed independently of any claim of ownership. Even when the
question of possession cannot be resolved without deciding the
Facts:
issue of ownership, the issue of ownership is to be resolved only
to determine the issue of possession. In his forcible entry complaint, petitioner Homer C. Javier,
represented by his mother and natural guardian Susan G.
Canencia, alleged that he is one of the sons of the late Vicente T.
Javier ,who was the owner of a 360-square meter (sq. m.) parcel
of land in Taytay Rizal. Petitioner’s family contented that since
Javier was born, he has lived in the residential house erected
thereon. Upon Vicente’s death, petitioner, together with his
mother, continued their possession over the same. On March 26,
2007, despite the petitioner’s vigorous objections, respondent
Susan Lumontad gained entry into the subject land and started to
build a two (2)-storey building (subject building) on a 150 sq. m.
portion thereof. No amicable settlement was reached, thus the
case of forcible entry was filed by petitioner against respondent.

Issue: W/N petitioner’s filing of forcible entry against respondent


is with merits.

Held:

Petitioner’s forcible entry complaint cannot be granted on its


merits, considering that he had failed to justify his right to the de
facto possession of the disputed premises. As pointed out by the
CA, the supposed document from which petitioner hinges his right
to the de facto possession of the subject land, only covers his
house and not the entire land itself. Nothing appears on record to
show that he has the right to the de facto possession of the
172.80 sq. m. portion which, on the contrary, appears to be
consistent with the claim of ownership of respondent in view of
TD No. OOTY-002-13031 covering the same property as


registered in her name. Thus, with no evidence in support of Santos was a good faith buyer after taking steps to verify the
petitioner's stance, and the counter-evidence showing identity of the seller. When she was showed the invoice, she
respondent's right to the de facto possession of the 172.80 sq. m. reasonably believed that he was a legitimate seller. With regard
portion as its ostensible owner, the forcible complaint must to unlawful deprivation, EDCA was not unlawfully deprived of the
necessarily fail. property by mere failure of consideration. There was already a
perfected contract of sale. Proof was even substantiated when
98. EDCA Publishing and Distributing Corp. v The Spouses EDCA gave the invoice as proof of payment upon delivery of the
Leonor and Gerardo Santos books. This did not amount to unlawful taking, because by the
GR No. 80298 delivery of EDCA to Cruz, ownership of the books already
transferred to him.
April 26, 1990
99
FACTS:
BPI Family Bank v Franco
G.R. No. 123498, November 23, 2007
An individual who identified himself as Professor Jose Cruz
placed an order through telephone with Edca Publishing. He
Facts:
ordered 400+ books and issued a personal check as payment.
Then he sold some of the books to Santos who, after verifying the
Franco opened 3 accounts with BPI with the total amount of 2M,
seller’s ownership from the invoice shown, paid Cruz. Meanwhile,
where such was traceable to a check issued by Tevesteco. The
Edca being suspicious over the second order placed by Cruz
funding for the 2M check was part of the P80M debited by BPI
verified with De La Salle College where he had claimed to be
from FMIC’s account (w/ deposit of P100M) and credited to
dean and was informed that no such person was under its
Tevesteco’s account pursuant to an Authority to Debit which was
employ. There was also no account with the bank against which
allegedly forged as claimed by FMIC. Tevesteco effected several
he had drawn his check. It was later found out that his real name
withdrawals already from its account amounting to
was Tomas de la Pena. Edca reported this to the police and
P37,455,410.54 including the P2M paid to Franco. Franco issued
through an entrapment, de la Pena was captured. They forced
two checks which were dishonored upon presentment for
their way inside Santos’ store and seized the books without any
payment due to garnishment of his account filed by BPI.
warrant.
BPI claimed that it had a better right to the amounts which
ISSUE: consisted of part of the money allegedly withdrawn from it by
Whether or not the owner was unlawfully deprived of the Tevesteco and ending up in Franco’s account. BPI urges us that
property? the legal consequence of FMIC’s forgery claim is that the money
transferred by BPI to Tevesteco is its own, and considering that it
HELD: No. was able to recover possession of the same when the money was
redeposited by Franco, it had the right to set up its ownership
thereon and freeze Franco’s accounts.


petitioners' property, has a potential direct access to Marcos
Issue: WON the bank has a better right to the deposits in highway. Petitoners filed a complaint for an Easement Right of
Franco’s account. Way and argue that it is unfair to require them to pay the value of
the affected road lots since the same is tantamount to buying the
Held: property without them being issued titles and not having the right
to exercise dominion over it.
No. Significantly, while Article 559 permits an owner who has lost
or has been unlawfully deprived of a movable to recover the exact
same thing from the current possessor, BPI simply claims
ownership of the equivalent amount of money, i.e., the value Issue: WON payment of indemnity in a right of way results to
thereof, which it had mistakenly debitedfrom FMIC’s account and alienation of subject lots.
credited to Tevesteco’s, and subsequently traced to Franco’s
account.
Held: No. Payment of the value of the land for permanent use of
Money bears no earmarks of peculiar ownership, and this the easement does not mean an alienation of the land occupied.
characteristic is all the more manifest in the instant case which Unlike in purchase of a property, should the right of way no longer
involves money in a banking transaction gone awry. Its primary be necessary and the servient estate demands that the easement
function is to pass from hand to hand as a medium of exchange, be extinguished, the value of the property received by the
without other evidence of its title. Money, which had been passed
servient estate by way of indemnity shall be returned in full to the
through various transactions in the general course of banking
business, even if of traceable origin, is no exception. dominant estate. This only reinforces the concept that the
payment of indemnity is merely for the use of the right of way and
not for its alienation.

-100
DEMETRIA DE GUZMAN v. FILINVEST DEVELOPMENT
CORPORATION
G.R. NO. 191710, 2015

Facts: Petitioners were co-owners of a parcel of land which is


enclosed and surrounded by other real properties belonging to
various owners. One of its adjoining properties is owned and
developed by respondent Filinvest(respondent) which, from



101 in the contract to sell, there contained a Restrictive Covenant
providing for some prohibitions regarding the expansion to the
Reyes vs. Spouses Ramos house which the owners would like to do with their house. The
February 11, 2015 G.R. No. 194488 controversy arose when the petitioner-spouses extended the roof
of their house to a point directly above the original front wall
FACTS: wherein respondent filed an action to demolish the unauthorized
Reyes filed a Complaint for easement of right of way against structures.
Spouses Ramos alleging that she was the registered owner of a
parcel of land Bulacan, and that the property used to be a portion ISSUE:
of Lot No. 3-B8 and was surrounded by estates belonging to (1) Whether or not the Restrictive Covenant is valid
other persons.Petitioner also alleged that respondents' 1,500- (2) Whether or not the structures built by the petitionerspouses
square-meter property surrounded her property, and that it was are legal and valid.
the only adequate outlet from her property to the highway. A 113-
square-meter portion of respondents' property was also the "point RULING:
least prejudicial to the respondents. The easement sought was 1. Yes. In a low cost socialized housing, owners developers are
the vacant portion near the boundary of respondents' other lot. constrained to build as many number of houses on a limited land
Petitioner insisted that her property was not isolated because of area to accommodate marginalized lot buyers, providing as much
her own acts. as possible the safety, aesthetic and decent living condition by
controlling overcrowding. There appears to be no cogent reasons
ISSUE:whether petitioner has the compulsory easement of right for not upholding restrictive covenants aimed to promote
of way over respondents' property aesthetics, health, and privacy or to prevent overcrowding.
DECISION: 2. Since extension constructed exceeds the floor area limits of the
NO. ART. 650 provides that: The easement of right of way shall Restrictive Covenant, petitioner-spouses can be required to
be established at the point least prejudicial to the servient estate, demolish the structure to the extent that it exceeds the prescribed
and, insofar as consistent with this rule, where the distance from floor area limits for as Article 1168 of the New Civil Code provides
the dominant estate to a public highway may be the shortest. that “when the obligation consists in not doing and the obligor
Based on the Ocular Inspection Report, petitioner's property had does what has been forbidden him, it shall be undone at his
another outlet to the highway. Furthermore, the convenience of expense”.
the dominant estate's owner is not the basis for granting an
easement of right of way, especially if the owner's needs may be
satisfied without imposing the easement. 103. Abellana v. CA, G.R. No. 97039, Apr. 24, 1992

Facts: The petitioners sued to establish an easement of right of


102. FAJARDO VS FREEDOM TO BUILD
way over a subdivision road which used to be a mere footpath
Facts: Freedom To Build, Inc., an owner-developer and seller of they and their ancestors had been using since time immemorial,
low-cost housing sold to petitioner-spouses a house and lot and and that, hence, they had acquired, through prescription, an


easement of right of way therein. The construction of a wall by the 105. QUINTANILLA V. ABANGAN G.R. No. 160613
respondents around the subdivision deprived the petitioners of February 12, 2008
the use of the subdivision road, which gives the subdivision
residents access to the public highway. They asked that the high FACTS:
concrete walls enclosing the subdivision and cutting off their A complaint for Easement of Right of Way was filed by
access to the subdivision road be removed and that the road be petitioner. This was dismissed by the trial court for failing to
opened to them. establish that the imposition of the right of way was the least
prejudicial to the servient estate. The RTC noted that there is
Issue: Whether or not the use of a footpath is a continuous
already a concrete fence around the area. Thus, substantial
easement.
damage and substantial reduction in area would be caused the
Held: The use of a footpath or road may be apparent but it is not servient estate.
a continuous easement because its use is at intervals and
ISSUE: WON the petitioner has the right of way against
depends upon the acts of man. It can be exercised only if man
respondents
passes or puts his feet over somebody else’s land.
RULING:
104. Bicol Agro-Ind vs Obias G.R. No. 172077 Oct. 09, 2009
NO. Petitioner failed to discharge the burden of proving the
Facts: existence and concurrence of all the requisites in order to validly
Bicol Sugar Development Corporation (BISUDECO) constructed claim a compulsory right of way against respondents.
a “disputed” road and the road was used by BISUDECO in To a legal easement of right of way, the following
hauling and transporting sugarcane to and from its mill site. requisites must be satisfied: (1) the dominant estate is
Respondents barricaded the disputed road and thus preventing surrounded by other immovables and has no adequate outlet to a
the petitioner’s vehicles from passing through. Petitioner alleged public highway; (2) proper indemnity has been paid; (3) the
that BISUDECO constructed the road pursuant to an agreement isolation was not due to acts of the proprietor of the dominant
with the owners of the ricefields that the road traversed. Petitioner estate; and (4) the right of way claimed is at the point least
contends that BISUDECO acquired a right of way over the prejudicial to the servient estate. The fourth requisite is absent.
properties of the landowners, which right of way in turn was The Court is in full accord with the ruling of the CA.
acquired by the petitioner when it bought BISUDECO’s assets.
106. QUIMEN V. CA- Easement
Held:
FACTS:
In order for the petitioner to acquire the disputed road as an
easement of right-of-way, it is incumbent upon them to show its A portion of the lots behind Anastacia’s were sold by her (as her
right by title or by an agreement with the owners of the land that brother’s administrator) brother to Yolanda who constructed a
said land traversed. house thereon despite having no access to the public road.



Yolanda was assured that Anastacia would give her a right of way the easement is not necessary on the ground that the dominant
on her adjoining property for p200 per square meter. estate now has an adequate access to a public road.
Anastacia barred her from passing through her property. Issue: Whether or not the Memorandum of Encumbrance can be
cancelled?
Yolanda filed an action with the proper court praying for a right of
way through Anastacia’s property. Held: No, the opening of an adequate access to a public road
extinguishes only legal or compulsory easements, not voluntary
ISSUE:
easements. This easement of right of way, like any other contract,
Whether or not the valid grant of an easement and the right of could be extinguished only by mutual agreement or by
way proposed by Yolanda is the least onerous/least prejudicial to renunciation of the owner of the dominant estate. (Art. 631, NCC)
the parties
108. Linda Rana vs Teresita Lee Wong, Sps. Ong, and Sps.
HELD: YES Uy June 30, 2014 G.R. No. 192861
The conditions for a valid grant of an easement of right of way Facts:
were clearly present. The evidence clearly shows that the Wong, et al availed of the remedy of judicial abatement of
property of private respondent is hemmed in by the estates of damages against Sps. Rana, claiming that the elevated and
other persons including that of petitioner; that she offered to pay cemented subject portion of the latter’s property affected their
P200.00 per square meter for her right of way as agreed between usual manner of ingress and egress, and that the subject
her and petitioner; that she did not cause the isolation of her backfilling may cause sudden collapse of land, and thus should
be declared “nuisances” which curtailed their use and enjoyment
property; that the right of way is the least prejudicial to the of their properties.
servient estate.
Issues:
WON the cemented subject portion and subject backfilling
107. Unisource Commercial and Development Corporation v. constituted a nuisance per se and thus should be abated
Chung et,al.
GR. No. 173252 July 17, 2009 Ruling:
Facts: No, the subject portions are not nuisances per se. It is not
injurious to the health or comfort of the community, as defined
A memorandum of encumbrance of a voluntary easement under Art. 694 of the Civil Code. It was primarily built to facilitate
entered in favor of Hidalgo was annotated at the back of the OCT the ingress and egress of Sps. Rana. But for introducing a
of Sandico. Sandico sold his land to herein petitioner carrying nuisance per accidens that particularly transgressed the aforesaid
over the encumbrance on the Transfer Certificate Title. Hidalgo rights, Sps. Rana should be liable for nominal damages.
also sold his land to herein respondents Chung. The petitioner
then filed a petition to cancel the Encumbrance contending that 109. AQUINO v MUNICIPALITY OF MALAY, AKLAN


Telmo is found administratively guilty of discourtesy in the course
FACTS: The Municipal Mayor of Aklan issued EO 10 to close and of official duties.
demolish the Boracay West Cove Hotel for the reason that the
construction violates the “no-build zone” of the Municipal
Ordinance which prohibits constructions 25 meters from the edge 111. EMILIO GANCAYCO VS CITY GOVERNMENT OF
of the mean high water mark. QUEZON CITY AND MMDA

ISSUE: WON the property is a nuisance per se FACTS:

HELD: No. The Court ruled that the property involved cannot be Justice Gancayco sought to be exempted from the ordinance
classified as a nuisance per se which can therefore be summarily requiring that the arcade is to be created by constructing the wall
abated. It is merely the hotel’s particular incident- its location- of the ground floor facing the sidewalk a few meters away from
because it was constructed in a no build zone and not its inherent the property line. Decades later, MMDA conducted operations to
qualities that rendered it a nuisance. However, the hotel is still a clear obstructions along EDSA, in consequence, they sent a
nuisance per accidens. Hence, the LGU may nevertheless notice of demolition to Justice Gancayco alleging that a portion of
properly order the hotel’s demolition in the exercise of its his building violated the National Building Code.
police power and the general welfare clause.
ISSUE: WON MMDA can legally demolish the property of Justice
110. BUSTAMANTE VS TELMO Gancayco.

Facts: HELD:
A complaint was filed by Bustamante against Telmo, whose land NO. MMDA has illegally demolished the property of Justice
encroached upon former’s lot. Subsequently, Bustamante put up Gancayco.
concrete poles on his lot. However, Telmos and their men
allegedly destroyed the concrete poles. The authority to order the demolition of any structure lies with the
Building Official. The penalty prescribed by the Ordinance itself
ISSUE: WON the concrete poles were nuisance per se does not include the demolition of illegally constructed buildings
Held: in case of violations. Instead, it merely prescribes a punishment.
The ordinance itself also clearly states that it is the regular courts
NO. A nuisance per se is that which affects the immediate safety that will determine whether there was a violation of the ordinance.
of persons and property and may be summarily abated under the
undefined law of necessity. Evidently, the concrete posts 112. PEREZ V. MADRONA G.R. No. 184478 March 21, 2012
summarily removed by Telmo did not at all pose a hazard to the
safety of persons and properties, which would have necessitated FACTS: Respondent-spouses built a house which encroached on
immediate and summary abatement. the sidewalk and that is in violation of PD 1096 of the National
Building Code and RA 917 on Illegally occupied/constructed
improvements within the road right-of-way. Petitioner sent a letter,


giving respondents ten days from receipt thereof to remove the -versus-
structure allegedly protruding to the sidewalk and showing that Federico Daclan, et al.
the respondents’ fence is a nuisance per se.
G.R. No. 197115 23 March 2015
ISSUE: W/N respondents structure is a nuisance per se
HELD: No. The structure is not a nuisance per se that presents
immediate danger to the communitys welfare. If petitioner
believes that respondents fence indeed encroaches on the Republic of the Philippines-versus- Federico Daclan, et al.
sidewalk, it may be so proven in a hearing conducted for that G.R. No. 197115, 23 March 2015
purpose. Not being a nuisance per se, but at most a nuisance per (Donation with conditions)
accidens, its summary abatement without judicial intervention is
unwarranted.
113. CHARLES BUMAGAT, et al. v. REGALADO ARRIBAY FACTS:
G.R. No. 194818, 9 June 2014 In 1972, the respondents donated land in La Union to the
government to establish the Agoo Breeding Station. Subject to
FACTS: Petitioner filed a complaint for forcible entry against the a condition that in case of non-use, abandonment or
Arribay alleging that with the aid of armed goons and through the cessation of the activities of the Bureau of Animal Industry (BAI),
use of intimidation and threats of physical harm, the latter entered possession or ownership shall automatically revert to the donor,
the former’s parcels of land and ousted them from their lawful including all permanent improvements. After the donation, the La
possession. Arribay sought for the dismissal of the complaint, Union Medical Center was constructed on 1.5-hectare portion of
claiming that the subject properties are agricultural lands – which the donated property. In 2003, the respondents demanded the
renders the dispute an agrarian matter and subject to the return of their donated lands on the ground that the breeding
exclusive jurisdiction of Department of Agrarian Reform station has ceased operations and the land has been abandoned.
Adjudication Board (DARAB).
ISSUE: Is the dispute within the jurisdiction of DARAB? ISSUE: Whether the property can be reverted to the
respondents-donors.
RULING: No. The CA failed to realize the fact that as between the
parties, there is no tenurial arrangement, not even an implied HELD:
one. For the DARAB to acquire jurisdiction over the case, there No. The preponderance of evidence points that the
must exist a tenancy relation between the parties. breeding station remained operational even after its transfer from
114 the Republic to the Province. The court cannot validly order the
Republic of the Philippines, represented by the Secretary of return the donated 1.5-hectare portion where the LUMC is
Agriculture situated to the respondent because they admitted that the 1.5-
hectare portion where the LUMC is constructed does not form


part of the lands they donated to the government, but mandatory according to Article 748 and non compliance results in
belonged other donors who are not parties to the instant case. a void donation.

116. REPUBLIC VS GUZMAN GR No. 132964, Feb. 18, 2000


– 115
Esperanza C. Carinan vs Spouses Gavino Cueto and FACTS: David Rey Guzman, a natural-born American citizen, is
Carmelita Cueto the son of the spouses Simeon Guzman (naturalized American)
GR No. 198636 and Helen Meyers Guzman (American citizen). In 1968, Simeon
October 8, 2014 died leaving to his heirs, Helen and David, an estate consisting of
Facts: several parcels of land in Bulacan.
Respondent Spouses Cueto alleged that In 1970, Helen and David executed a Deed of Extrajudicial
the petitioner and her husband acquired the rights to a parcel of Settlement of the Estate, dividing and adjudicating to themselves
land under the name of the GSIS and were to assume payment of all of the property, and registered it to the Register of Deeds a
the monthly amortizations. Respondent paid the total obligation to year after.
GSIS along with the payment for transfer and renovation of the
residential house since Petitioner, who was widowed by then, In 1981, Helen executed a Deed of Quitclaim, assigning,
could not pay the amortizations. The petitioner and Jazer, the transferring and conveying her ½ share of the properties to David.
petitioner’s son, executed a Deed of Absolute Sale with the But since it was not registered, she executed another Deed of
condition to buy the land back within three years. The petitioner Quitclaim to confirm the first.
surrendered the TCT to the respondents. In 1994, Atty. Batongbacal wrote the Office of the Solicitor
Respondents demanded from the petitioner to fulfill the transfer General and furnished it with documents showing that David’s
of the property but they did not comply. Petitioner countered ownership of ½ of the estate was defective. He argued that Art.
saying that it was donation and there was no written or oral XII of the Constitution only allows Filipinos to acquire private
agreement for transfer or repayment. lands in the country. The only instances when a foreigner may
Issue: acquire private property are by hereditary succession and if he
was formerly a natural-born citizen who lost his Filipino
Whether or not the agreement was a contract of loan or donation. citizenship. Moreover, it contends that the Deeds of Quitclaim
Ruling: executed by Helen were really donations inter vivos.

The SC ruled that the purely gratuitous act was not supported by Republic filed with RTC a Petition for Escheat praying that ½ of
records citing the Civil Code that any donation over Php500 shall David’s interest be forfeited in its favor. RTC dismissed. CA
be in writing otherwise it is void. The formal requirements are affirmed.
ISSUE: Whether or not there was a donation inter vivos.


Mine: whether or not there was repudiation in the case at bar. FACTS: Branoco claimed ownership over a property through
purchase from Rodriguez, who had acquired the said property
HELD: No. Not all the elements of a donation are present. The
from Rodrigo by way of donation. Rodrigo subsequently sold the
transfer of the properties by virtue of a Deed of Quitclaim resulted
same property to Vere.
in the (1) reduction of Helen’s patrimony as donor and the (2)
consequent increase in the patrimony of David as donee.
However, Helen’s (3) intention to perform an act of liberality in
ISSUE: WON the contract of donation of Rodriguez and Rodrigo
favor of David was not sufficiently established. The 2 Quitclaims
was effectively cancelled when Rodrigo sold the property to Vere.
reveal that Helen intended to convey to her son certain parcels of
land and to re-affirm it, she executed a waiver and renunciation of
her rights over these properties. It is clear that Helen merely
contemplated a waiver of her rights, title, interest over the lands in HELD: No. When the donor used the words that the gift "does not
favor of David, not a donation. She was also aware that donation pass title during my lifetime; but when I die, she shall be the true
was not possible. owner of the two aforementioned parcels"] the donor meant
nothing else than that she reserved of herself the possession and
Moreover, the essential element of acceptance in the proper form usufruct of said two parcels of land until her death, at which time
and registration to make the donation valid is lacking. The SPA the donee would be able to dispose of them freely.
executed by David in favor of Atty. Abela was not his acceptance,
but an acknowledgment that David owns the property referred to 118
and that he authorizes Atty. Abela to sell the same in his name. Central Philippines University vs C.A.
Further, there was nothing in the SPA to show that he indeed
accept the donation. Facts:
• In 1939, late Don Ramon Lopez. Sr. executed a deed of
There is NO valid repudiation of inheritance as Helen had already donations in favor of Central Philippines University (CPU)
accepted her share of the inheritance when she, together with of a parcel of land with the following conditions:
David, executed a Deed of Extrajudicial Settlement of the Estate, • The land shall be exclusively use for the establishment of
dividing and adjudicating between them all the properties. By
and use of a medical college.
virtue of that settlement, the properties were registered in their
• CPU shall not sell, transfer or convey to any third party or
names and for 11 years, they possessed the land in the concept
of owner. Thus, the 2 Quitclaims have no legal force and effect. any way the land.
Helen still owns ½ of the property. • The said land shall be called “Ramon Lopez Campus”
• However in 1989, the respondents who are heirs of Don
117
Ramon filed an action for annulment of donation, re-
Villanueva vs Branoco conveyance and damages against CPU alleging that since
1939 has not complied with the conditions of the donation



and that the University negotiate with National House obligations itself cannot be demanded until the court
Authority (NHA) to exchange the donated land with has fixed the period for compliance therewith and
another land. such period has arrived.
ISSUE: Ø However, this general rule cannot be applied in this case.
Whether or not all the rights of the done are deemed lost More than a reasonable period of 50 years has been
and extinguished because of the non-fulfillment of the conditions? allowed for petitioner to avail of the opportunity to comply
with the obligations
SUPREME COURT RULING: Ø Art. 1191 of Civil Code, when one of the obligators
• Supreme Court finds that since the records are clear and cannot comply with what is incumbent upon him, the
facts are undisputed that since the execution of the deed oblige may seek rescission and the court shall decree
of donation until filing of instant action, petitioner has failed the same unless there is just cause authorizing the
to comply with its obligation as done. CPU has slept its fixing of the period
obligation for unreasonable length of time so it is only just
and equitable to declare the donation ineffective. – 119
• Supreme Court declared that the conditions set forth in the
donation were resolutory conditions which mean the
THE INSULAR LIFE v. EBRADO and PASCUALA VDA. DE
fulfillment of the conditions extinguishes the obligation.
EBRADO

Ø Under Art. 1181 of the Civil Code, in conditional G.R. No. L-44059, October 28, 1977
obligations, the acquisition of rights, as well as
extiguishment or loss of those already acquired shall (insurance proceeds; common law wife)
depend upon happening of the event which
constitutes the condition. Thus, when a person donated Facts:
Cristor Ebrado was issued by The Life Assurance Co., Ltd., a
a land to another on the condition that the order would policy with a rider for Accidental Death. He designated Carponia
build on the land a school. The condition imposed was T. Ebrado as the revocable beneficiary in his policy. He referred
not a condition precedent or suspensive condition but to her as his wife. Cristor was killed by a failing branch of a tree.
a resolutory condition. Insular Life was made liable to pay the coverage. Carponia T.
Ø When obligations does not fix a period from its nature and Ebrado, who was found to be a common law wife, filed with the
circumstances it can be inferred that the period was insurer a claim for the proceeds. Pascuala Vda. de Ebrado also
filed her claim as the widow of the deceased insured and asserts
intended, the general rule provided in Art. 1197 of the Civil
that she is the one entitled to the proceeds. The lower court
Code applies, which provides that the court may fix the declared Carponia as disqualified.
durations thereof because the fulfillment of the


Issue: WON automatic reversion of the land to the donor is a valid
WON a common-law wife named as beneficiary in the life condition in a deed of donation
insurance policy of a legally married man can claim the proceeds.
HELD:
Held: YES. Such stipulation, not being contrary to law, morals, good
customs, public order or public policy, is valid and binding on the
No. When not provided for by the Insurance Law, the contract of
parties to the donation. When a deed of donation, expressly
life insurance is governed by the general rules of the civil law
provides for automatic revocation and reversion of the property
regulating contracts. Article 2012 of the same Code, any person donated, the rules on contract should apply, and not Article 764 of
who is forbidden from receiving any donation under Article 739 the Civil Code. Since Article 1306 of said Code authorizes the
cannot be named beneficiary of a life insurance policy by the parties to a contract to establish such stipulations, . . . not
person who cannot make a donation to him. Common-law contrary to law, . . . public order or public policy, we are of the
spouses are barred from receiving donations from each other opinion that, at the very least, that stipulation of the parties
providing for automatic revocation of the deed of donation,
– 120 without prior judicial action for that purpose, is valid.

ZAMBOANGA V PLAGATA
G.R. No. 148433
September 30, 2008
FACTS:
ZBTKBI entered into a Deed of Donation whereby it donated to
the Republic a parcel of land with a condition that in the event
barter trading shall be phased out, prohibited, or suspended for
more than one (1) year in Zamboanga City, Philippines, the
parcel of land shall revert back to the DONOR without need of
any further formality or documentation.
On 17 June 1988, the Office of the President issued
Memorandum Circular No. 1 which totally phased out the
Zamboanga City barter trade area.
ISSUE:



Metropolitan Fabrics Inc. and Enrique Ang vs. Prosperity
Credit Resources Inc., Domingo Ang and Caleb Ang
121. QUIJADA VS. CA, G.R. 154390. 17 March 2014
G.R. NO. 126444, DEC. 4, 1998
TOPIC: Action to Assail Mortgage
(donation with a condition; sale of donated property)

Facts: Trinidad Quijada, together with her siblings executed a FACTS: 'In July 1984, MFI obtained a loan from PCRI in the
Conditional Deed of Donation in favor of the municipality of amount of P3.5M, represented by herein respondents Domingo
Talacagon, the condition is being that the land shall be used and Caleb Ang. The blank loan forms had no entries specifying
exclusively for the construction of a provincial high school. the rate of interest and schedules of amortization. Petitioner
Trinidad remained in possession of the Land. Thenafter, Trinidad Enrique and Vicky Ang, entrusted to the respondents their seven
sold the land to Mondejar. The heirs to Trinidad (herein (7) titles to secure the P3.5M loan. In September 1984, It was
Petitioners) filed a complaint for forcible entry against the then that the petitioners learned that PCRI had filled up the said
respondents. The proposed campus did not materialized and the checks with dates and amounts reflected at 35% interest rate per
Sangguniang Bayan enacted a resolution donating back the land annum, instead of just 24%, and a two-year repayment period,
to the donor. In the meantime, Mondejar conveyed portions of the instead of 10 years.
land to complaint for quieting of the title recovery of possession
and ownership of the land.
ISSUE: Whether or not the action to assail the mortgage already
Issue: W/N the sale between Trinidad and Mondejar is valid. prescribed.

Held: No. The Donor may have an inchoate interest in the


donated property during the time that ownership of the land has HELD:
not reverted to her. Such inchoate interest maybe the subject of
contracts including a contract of sale. In this case, what the donor YES. Here, it is important to first determine if the
sold was the land itself which she no longer owns. It would have mortgage was void or merely voidable. Where consent was
been different if the donor-seller sold her interest over the given through fraud alone, the contract was voidable, not void ab
property under the deed of donation which is subject to the initio. Article 1390, in relation to Article 1391 of the Civil Code,
possibility of reversion of ownership arising from the non- provides that if the consent of the parties was obtained
fulfillment of the resolutory condition. through fraud, the contract is considered voidable and may
be annulled within four (4) years from the time of the
discovery thereof. Thus, because the mortgage involving the
seven lots was registered on September 5, 1984, they had until
CASE #122


September 5, 1988. But their complaint was instituted in the RTC Torrens system and not to unregistered lands, such as the
on October 10, 1991. Hence, the action had already prescribed. property in the suit; and (2) the principle is inapplicable to banking
institutions who are called upon to exercise greater care and
prudence before entering into a contract of mortgage.
124

123 SPOUSES FRANCISCO SIERRA vs.


PAIC SAVINGS AND MORTGAGE BANK, INC
Rural Bank of Cabadbaran v Melecio-Yap [GR 178451]
G.R. No. 197857 September 10, 2014
FACTS:
FACTS:
Erna Melecio-Yap and her 5 siblings inherited a lot from their
parents. Said lot had been a subject of a real estate mortgage Goldstar Conglomerates, Inc. (GCI), obtained a loan from Summa
following a loan procured from petitioner RBCI, as authorized by Bank, now respondent Paic Savings and Mortgage Bank, Inc.
a notarized SPA purportedly executed by Erna’s parents. Upon (PSMB). As security therefor, GCI executed in favor of PSMB six
default in payment, the lot was sold to RBCI as the highest bidder (6) promissory notes and a Deed of Real Estate Mortgage. As
in the public auction sale. Respondents claimed that they were additional security, petitioners Sierra mortgaged four (4) parcels
not aware of the loan obtained by Erna and did not authorize the of land in Antipolo. GCI defaulted in the payment of its loan to
mortgage transaction, and that the SPA bore their falsified PSMB. The properties were foreclosed and new TCT were issued
signatures. RBCI claims to be a mortgagee in good faith. to PSMB since petitioners failed to redeem the subject properties
within the redemption period.
ISSUE:
ISSUE: WHETHER OR NOT PETITIONERS ARE MERE
W/N the subject property was validly mortgaged and foreclosed ACCOMODATION MORTGAGORS
W/N RBCI was a mortgagee in good faith
HELD: Yes. An accommodation mortgagor is a third person who
HELD: No on both issues. The forged SPA is sufficient grounds to is not a debtor to a principal obligation but merely secures it by
render the mortgage null and void, insofar as the shares of the mortgaging his or her own property. Petitioners’ claim of lack of
other co-owners, whose consent thereto were not actually "proper instruction on the intricacies in securing [the] loan from
obtained, are concerned. Thus, RBCI may validly own, by virtue the bank" is belied by the fact that petitioners Francisco and
of the foreclosure proceedings, only that portion of the property Rosario Sierra had previously mortgaged two (2) of the subject
belonging to Erna. properties twice to the Rural Bank of Antipolo. Moreover,
petitioners did not: (a) demand for any loan document containing
The RBCI cannot be considered a mortgagee in good faith since the details of the transaction, and (b) offer to pay the purported
(1) such doctrine applies only to lands registered under the partial loan proceeds they received at any time.


The Court finds the element of fraud to be wanting; hence, there
can be no unfair competion.
125

Shang Proper,es v. St. Francis Development Corpora,on GR


126
No. 190706; July 21, 2014
WILLAWARE PRODUCTS CORPORATION VS. JESICHRIS
FACTS: Respondent, a domestic corporation, engaged in real MANUFACTURING CORPORATION
estate business, used the mark “ST. FRANCIS” to identify its (G.R. No. 195549, September 3, 2014)
numerous property development projects, such as a shopping Facts:
mall called the “St. Francis Square,” and a mixed-use realty Jesichris Manufacturing Company filed this present complaint for
project plan that includes the St. Francis Towers. The respondent damages for unfair competition to enjoin Willaware Products
Corporation the petitioner from manufacturing and distributing
claimed that petioners could not have the mark “THE ST. plastic-made automotive parts similar to Jesichris Manufacturing
FRANCIS TOWERS” registered in their names, and that Company. The respondent alleged that it is a duly registered
petioners’ use of the marks “THE ST. FRANCIS TOWERS” and partnership engaged in the manufacture and distribution of plastic
and metal products. Since its registration in 1992, respondent
“THE ST. FRANCIS SHANGRI-LA PLACE” in their own real
company has been manufacturing in its Caloocan plant and
estate development projects constitutes unfair competion as well distributing throughout the Philippines plastic-made automotive
as false or fraudulent declaration. parts while petitioner corporation which is engaged in the
manufacture and distribution of kitchenware items made of plastic
ISSUE: and metal has its office near that of the Respondent company.
Respondent further alleged that due to the physical proximity of
petitioner’s office to respondent’s office, and in view of the fact
WON petioners are guilty of unfair competition in using the marks
that some of the respondent’s employees had transferred to
“THE ST. FRANCIS TOWERS” and “THE ST. FRANCIS petitioner, petitioner had developed familiarity with respondent’s
SHANGRI-LA PLACE.” products, especially its plastic-made automotive parts.

RULING:
Issue:
NO. The unfair competion concept refers to a "Passing off" where 1. W/N there is unfair competition under human relations when
the defendant gives his goods the general appearance of the the parties are not competitors and there is actually no damage
goods of his competitor with the intention of deceiving the public on the part of Jesichris?
that the goods are those of his competitor.” The The test is
whether the acts of the defendant have the intent of deceiving" .



ABS-CBN and Reuters but also its staff had no knowledge that it
Held: aired petitioner’s footage. On the 3rd of December 2004, the
Assistant City Prosecutor found probable cause to indict the staff
(1)Yes, petitioner is guilty of unfair competition under Art, 28 of of GMA namely, Dela Pena and Manalastas. Consequently, the
the CC. From the foregoing, it is clear that what is being sought to respondents appealed the Prosecutor’s resolution before the
be prevented is not competition per se but the use of unjust, DOJ, and the latter ruled in favor of the respondents and held that
oppressive or high handed methods which may deprive others of
good faith may be raised as a defense in the case. Meanwhile,
a fair chance to engage in business or to earn a living.
DOJ Acting Secretary Alberto C. Agra issued a resolution which
Plainly,what the law prohibits is unfair competition and not
competition where the means use dare fair and legitimate. reversed Sec. Gonzales’s resolution and found probable cause to
charge Dela Pena, Manalastas, as well to indict Gozon, Duavit
To add, the instant case falls under Article 28 of the Civil Code on Jr., Flores, and Soho. Subsequently, Court of Appeals granted
human relations and not unfair competition under Republic Act the Petition for Certiorari to reverse and set aside the resolution
No. 8293, as the present suit is a damage suit and the products of the Acting DOJ and granted the issuance of temporary
are not covered by patent registration. If the products are covered restraining order and/or Writ Preliminary Injuction.
under said patent registration, such patent may be considered as
an intellectual property right. A fortiori, the existence of patent ISSUE
registration is immaterial in the present case. Whether or not there is probable cause to find respondents to be
held liable criminally for the case of copyright infringement under
127 G.R. No. 195956, 1 March 2015
the Intellectual Property Law (RA 8293, as amended)?
FACTS HELD
A criminal complaint was filed by ABS-CBN against GMA for The petition of ABS-CBN was partially granted by the Supreme
violating Sections 177 and 211 of the Intellectual Property Code, Court. An order was directed to the Q.C. Branch 93 to continue
as amended by RA 8293. This was because respondent GMA the proceedings against the staff of GMA, Dela Pena and
aired footage of the arrival and homecoming of Angelo Dela Cruz Manalastas due to copyright infringement. The other
from Iraq at NAIA without the consent of the petitioner. The respondents, Gozon, Duavit Jr., Flores, and Soho were held not
petitioner claimed that there was an agreement between them liable for the criminally; act of copyright infringement. It was held
and Reuters in exchange of the latter’s news and video material. that their membership in GMA’S Board of Directors doesn’t mean
Additionally, Reuters ensured the petitioner the latter’s materials that they have knowledge, approval, or participation in the
cannot be aired in the country. GMA was a subscriber of Reuter’s criminal act, as there is a need for the direct or active participation
and CNN live feeds. After having received the live feed of Angelo in such act. Also, there was lack of proof they actively participated
Dela Cruz’s arrival and homecoming, it immediately aired the or exercised moral ascendancy over Dela Pena and Manalastas.
video from that news feed. However, the respondent not only
alleged its news staff was not aware of the agreement between


The Court held that the respondent’s mere act of rebroadcast of -128
petitioner’s news footage without the latter’s authority creates
Taiwan Kolin vs Kolin Electronics
probable cause to find the former’s personnel criminally liable for
violating the provisions of Intellectual Property Code, particularly GR 209843 March 25, 2015
Section 216-217 of RA 8293, as amended, since both have not
exercised diligence in their function to prevent that the footage Facts:
from being aired on television. They were aware there would be Taiwan Kolin Corp. sought to register the trademark “KOLIN” in
consequences in carrying ABS-CBN footage in their broadcast. Class 9 goods. Kolin Electronics opposed the application on the
ground that the trademark “KOLIN” is identical, if not confusingly
similar, with its registered trademark “KOLIN”. The latter argeud
that the products are not only closely-related because they fall
under the same classification, but also because they are
inherently similar for being electronic products and are plugged
into electronic sockets and perform a useful function.

Issue:
W/N the products are closely related

Held:
No. To confer exclusive use of a trademark, emphasis should be
on the similarity or relatedness of the goods and/or services
involved and not on the arbitrary classification or general
description of their properties or characteristics.
Products classified under Class 9 can be further classified into 5
categories. Accordingly, the goods covered by the competing
marks between Taiwan Kolin and Kolin Electronics fall under
different categories. Taiwan Kolin’s goods are categorized as
audio visual equipments, while Kolin Electronics’ goods fall under
devices for controlling the distribution and use of electricity. Thus,
it is erroneous to assume that all electronic products are closely


related and that the coverage of one electronic product
necessarily precludes the registration of a similar mark over
another.

You might also like